You are on page 1of 47

Reports for

iCAT 01 (2014
Pattern)

Overview Accuracy

Time Allocation

Solution Question
Key
Selection

Unattempted Questions
Report

National
Percentile

Collapse All

Section I
1.

3 Marks

If f(x, y) = 4x5 5x4y + y5, for which of the following ranges is f(x, y) always non-negative?
1)

x > 4, y < 5/2

2)

x > 4, y < 5/2

3)

x > 2, y > 4

4)

x > 0, y > 3

Solution:
Consider the expression 4x5 + y5
4x5 + y5 = x5 + x5 + x5 + x5 + y5
By the AM-GM inequality,
(x5 + x5 + x5 + x5 + y5)/5 (x20y5)1/5 for non-negative x and y.
4x5 + y5 5x4y
f(x, y) = 4x5 5x4y + y5 0
f(x, y) 0 for positive x and y.
Only in case of option 4, x and y are always positive.
Hence, option 4.
2.

3 Marks

Ramas husbands age is obtained by reversing the digits of her age. Her husbands age is greater than her age by 1/11th of sum of their
ages. Find the sum of their ages.
1)

110

2)

96

3)

99

4)

77

Solution:
The difference between a number and number obtained by reversing the digits of the number is always divisible by 9.
Sum of the ages = 11 Difference
RHS is divisible by 11 as well as 9. So, LHS i.e., the sum is also divisible by 99 (= 11 9).
The only option that satisfies this is option 3.
Hence, option 3.
Alternatively,
Let Ramas age be 10x + y.
Husbands age = 10y + x
10y + x (10x + y) = 1/11 (10x + y + 10y + x)
9(y x) = x + y
10x = 8y
As x and y are single digit numbers, x = 4 and y = 5
Sum of their ages = 11(x + y) = 11 9 = 99
Hence, option 3.

http://testfunda.com/LMS/Student/NewReports.aspx

1/47

3.

A train running between Delhi and Agra passes 10 stations en-route. In how many ways can it stop at three of these ten stations such
that no two of them are consecutive?

3 Marks

1)

45

2)

56

3)

112

4)

120

Solution:
If we remove the 3 stations at which the train will stop we will have 7 stations left.
There will be 6 gaps in between the 7 stations and 2 at the end, therefore, 8 places in which the 3 stations at which the trains stop can be
arranged.
The three stations at which the train stops can be arranged in these 8 gaps in 8C3 = 56 ways.
Hence, option 2.
Group Question
Answer the following question based on the information given below.
The chart below shows the marks obtained by three students Palak, Aastha and Shreya in a class test of maximum marks 35 in Mathematics
(M), Economics (Eco), Business Studies (B.S.), English (Eng), Accountancy (Acc) and Physical Education (P.E.).

4.

3 Marks

What is the average score of the three girls in Business Studies?


1)

25

2)

28

3)

30

4)

35

Solution:
Marks scored by Palak, Aastha and Shreya in Business Studies are 25, 35 and 30 respectively.

http://testfunda.com/LMS/Student/NewReports.aspx

2/47

Hence, option 3.
5.

Who among the following got the highest percentage of marks?

3 Marks

1)

Shreya

2)

Aastha

3)

Palak

4)

All have equal percentages

Solution:
The marks obtained by each in all subjects are:

Since, Shreya has the highest total, she has the highest percentage.
Hence, option 1.

6.

Grade 'A' is awarded to a student in a subject, if she obtains more marks than the average marks in Accountancy. In how many of
the subjects did Palak get grade 'A'?

3 Marks

1)

2)

3)

4)

Solution:
Average marks in Accountancy = (20 + 30 + 35)/3 = 28.33
So, grade 'A' is awarded if a student obtains more than 28.33 marks in any subjects.
Palak scored more than 28.33 marks in Mathematics, Economics and Physical Education.
Hence, option 3.
7.

3 Marks

p is the root of the equation x2 + cx + d = 0


0 < d c 1. Which of the following best describes p?
1)
2)

3)

4)

Solution:
Let p and q be the roots of the equation.
p + q = c, and pq = d

http://testfunda.com/LMS/Student/NewReports.aspx

3/47

Now, 0 < d c 1
c and d are both positive.
Since product of the two roots is positive and their sum is negative, both p and q are negative.
|p| + |q| = |c|
|p| |c|
|p| |c| 1
|p| 1
Hence, option 1.
8.

3 Marks

Two sides of ABCD are parallel and its diagonals are equal. The two parallel sides are of lengths 12 cm and 10 cm respectively, while
the length of one of its non-parallel sides is 7 cm. What is the length of its diagonals?
1)

11 cm

2)

12 cm

3)

13 cm

4)

14 cm

Solution:
As the diagonals of the quadrilateral are equal and two of its sides are parallel, the quadrilateral will be an isosceles trapezium.
Length of its two non-parallel sides will be equal.

Draw AM and BN parallel to CD.


Using the Pythagoras theorem in AMD,

Using the Pythagoras theorem in AMC,

Length of diagonals = 13 cm
Hence, option 3.
Alternatively,
By Ptolemy's theorem, if a quadrilateral is inscribed in a circle, then the sum of the products of its two pairs of opposite sides is equal to
the product of its diagonals.
Also, every isosceles trapezium is cyclic.
Applying Ptolemy's theorem to the isosceles trapezium, we get,
AC BD = AB CD + BC AD
The diagonals are equal, we have,
AC2 = 12 10 + 7 7 = 169
AC = 13 cm
Hence, option 3.

9.

3 Marks

In an exam, there are 7 tests of various subjects. Each test is of 50 marks. Gajanan scored 40% marks altogether. His average marks in
Mathematics, English and Physics were 24 and highest marks among these three subjects were 32. The passing marks in any subject
were 17.5. Find the difference between maximum and minimum number of subjects in which Gajanan could have failed.
1)

http://testfunda.com/LMS/Student/NewReports.aspx

4/47

2)

3)

4)

Solution:
Total marks = 40% of 350 = 140
Gajanan scored 32, x and y in Mathematics, English and Physics respectively.
32 + x + y = 24 3 = 72
x + y = 40
Maximum (Mathematics, English, Physics) = 32
Thus, he can either pass in all the three subjects (i.e., Mathematics, English, Physics) or fail in one subject and pass in other two.
Score in other 4 subjects = 140 72 = 68
Now in the remaining 4 subjects, if he gets 17 in each, he fails in all the four subjects or he fails in only one subject and passes in the
remaining three subjects.
Thus, he can pass in at most 6 subjects and at least two subjects
Failmax = 5 and Failmin = 1
Required difference = 5 1 = 4
Hence option 4.
Group Question
Answer the following question based on the information given below.
In a five year integrated course in a college, only the students who pass the annual university examinations are promoted to the next year.
There is no lateral admission in the college and students are admitted only in the first year of the course.
The table below shows the number of students studying in different years in the last five years of the college.

In the last five years, 120 students have failed in the first year of the college. The pie-chart below shows the percentage of students who failed
in the each academic year in the first year of the college with respect to total students who failed in first year in the last five years.

10.

3 Marks

How many students join the college from the academic years 2011-12 to 2014-15?

http://testfunda.com/LMS/Student/NewReports.aspx

5/47

1)

623

2)

604

3)

592

4)

408

Solution:
Total students who failed in the first year of course in the last five academic years = 120
Number of first year student who failed in the given years are 36, 18, 24, 18, 24 respectively.
Number of students joining the college from 2011-12 to 2014-15
= (192 36) + (164 18) + (144 24) + (188 18) = 156 + 146 + 120 + 170 = 592
Hence, option 3.
11.
3 Marks

In how many of the course years did more than 10% students fail in 2013-14?
1)

2)

3)

4)

Solution:
Of 144 first year students in 2013-14, 18 students failed. Remaining 144 18 = 126 students were promoted to the second year in
2014-15.
Number of second year students in 2014-15 = 142
Number of second year students who failed in 2013-14 = 142 126 = 16 i.e., 10.13% of the second year students failed in
2013-14.
So, remaining 158 16 = 142 students were promoted to the third year in 2014-15.
Number of third year students in 2014-15 = 176
Number of second year students who failed in 2013-14 = 176 142 = 34 i.e., 15.6% of the third year students failed in 201314.
Continuing this way, we have
The number of students who passed or failed in 2013-14 are:

In all the course years except for the fourth year, more than 10% students have failed in each course year.
Hence, option 1.

12.

3 Marks

What was the percentage drop in the total number of students who passed in the year 2013-14 over 2012-13?
1)

12.1%

2)

11.7%

3)

10.5%

http://testfunda.com/LMS/Student/NewReports.aspx

6/47

4)

14.7%

Solution:
The number of students who passed in 2012-13 and 2013-14 can be calculated as per the previous question of this set. Thus,

Difference = 1010 904 = 106


The required percentage = (106/1010) 100 = 10.5%
Hence, option 3.
13.

3 Marks

A1and A2 are arithmetic progressions. First term of both the progressions is 72. For A1, the sum of the first 30 terms is equal to the sum
of the first 43 terms. If common difference of A2 is 1 less than that of A1, find the sum of first 7 terms of A2.
1)

441

2)

420

3)

399

4)

280

Solution:
For A1, let the common difference be d.
S30 = S43
t31 + t32 + + t43 = 0
t37 = 0
t37 = 72 + 36d = 0
d = 2
For A2, a = 72 and d = (2) 1 = 3
Hence, option 1.
14.

3 Marks

Saurabh is a Cricket fan and he started working with Cricdata as a writer. He was offered Rs. 70,000 plus a top quality cricket kit as his
annual remuneration. He left the company 5 months before the annual contract ends and was given Rs. 35,000 and the cricket kit as the
pro-rata based remuneration. How much was the cricket kit worth?
1)

Rs.13,000

2)

Rs.22,000

3)

Rs.14,000

4)

Rs.20,000

Solution:
Let price of the cricket kit be Rs. x.
Salary for 12 months = x + 70000
Salary for 7 months = x + 35000
Salary for 5 months = (x + 70000) (x + 35000) = 35000
So, monthly salary = Rs. 7,000
Annual salary = Rs. 84,000
Thus the kit is worth, 84000 70000 = Rs. 14,000

http://testfunda.com/LMS/Student/NewReports.aspx

7/47

Hence, option 3.
15.

3 Marks

A man wished to dig a well for irrigating his fields. On the first day, he dug two meters and on next day with the help of five workers, he
dug eight meters. On the third day with three additional men, he dug 12 meters. On the fourth day, with 3 additional men, he dug 16
meters. On the fifth day, with the help of 3 more men he dug 20 meters and so on. He took 25 days to completely finish digging the well.
How deep would the well have been at the end of the 25th day?
1)

1298 meters

2)

1402 meters

3)

1296 meters

4)

1396 meters

Solution:
Let D is the depth of the well.

D = 2 + 8 + 12 + 16 + 20 + ... + tn
Let S = 8 + 12 + 16 + 20 + ... + tn
where n = 24

Substituting a = 8, d = 4 and n = 24
S = 1296
D = 2 + S = 2 + 1296 = 1298 meters
Hence, option 1.
16.

3 Marks

RBI lends a certain sum of money to Yes Bank at repo rate 10% per annum for two years. Yes Bank then lends the same money to
Adanis Group at 10% per annum, compounded annually, for 2 years. Find the % profit made by Yes Bank in this transaction. (Consider
repo rate is the rate at which RBI lends money to a bank at simple interest.)
1)

2)

3)

4)

Solution:
Let RBI lends Rs.100 to Yes Bank for two years at 10% per annum.
So, interest expense of Yes Bank = 100 10% 2 = 20
Now, if Yes Bank lends the same amount at compound interest rate of 10% for two years:
A = P(1 + R)N
A = 100(1 + 0.1)2
A = 121
So, interest income of Yes Bank = 121 100 = 21
Net income of Yes Bank = Interest Income of Yes Bank Interest Expense of Yes Bank
= 21 20 = 1
Hence, percentage profit made by Yes Bank = (1/20) 100 = 5%
Hence, option 4.
17.

3 Marks

Two trains are 30 km apart and they start moving towards each other at the same time with 30 km/hr and 70 km/hr. At the same time, a
crow starts flying to and fro in between the two trains at a speed of 80 km/hr till the two trains meet each other. Find the total distance
covered by the crow.
1)

30 km

http://testfunda.com/LMS/Student/NewReports.aspx

8/47

2)

24 km

3)

27 km

4)

21 km

Solution:

Also, the crow moves between the two trains for 0.3 hours with a speed of 80 km/hr.
Distance covered by the crow in 0.3 hours = 0.3 80 = 24 km
Hence, option 2.
18.

3 Marks

If K = N (N2 + 2963) then, for N > 1000 which of the following numbers always divides K?
1)

2)

12

3)

18

4)

Solution:
N(N2 + 2963) = N(N2 + 2964 1)
= N(N2 1 + 2964)
= N(N2 1) + 2964N
= (N 1)(N)(N + 1) + 2964N
The product of three consecutive natural numbers is always divisible by 6.
2964 is a multiple of 3 and is even and hence is a multiple of 6.
N(N2 + 2963) is always a multiple of 6.
Hence, option 1.
19.

3 Marks

A point P(p, q) lies within a region M which is in the first quadrant and satisfies the condition 5x + 12y < 60, what is the probability that p
> 6?
1)

0.30

2)

0.33

3)

0.25

4)

0.67

Solution:
The line 5x + 12y = 60 intersects the x-axis at (12, 0) and the y-axis at (0, 5).
The region M in the first quadrant forms a right angle with base = 12 units, height = 5 units.
Area of this triangle = 30 sq. units

http://testfunda.com/LMS/Student/NewReports.aspx

9/47

The region formed by p > 6 and 5x + 12y < 60 is also a right triangle with base = 6 cm, height = 2.5 units.
Area of this triangle = 7.5 sq. units
Required probability = 7.5/30 = 0.25
Hence, option 3.

Group Question
Answer the following question based on the information given below.
In the year 1719, the quality of the software was decided on four factors: Design (D), Coding (C), Testing (T) and Documentation (DOC). The
ratings provided in each of the individual field are marked as A, B, C and D which have a score of 16, 8, 4 and 0 respectively. The overall
quality score (OQS) is the average of the scores in all four fields.

IBMs rating in design = TCSs rating in Documentation.


In documentation, Accenture's rating = CTS's rating
Cognizant's rating in testing = Wipro 's rating in coding
Overall rating of Congnizant = Overall rating of Wipro

20.

3 Marks

What is the overall quality score of Accenture?


1)

2)

3)

4)

11

Solution:
CTSs ratings are D, B and A.
Sum of the scores of three ratings = 0 + 8 + 16 = 24
CTSs overall quality score = 8
Sum of ratings = 8 4 = 32
Score of the fourth rating = 32 24 = 8
So, Accentures score in documentation = 8
Accentures other three scores are 16, 0, 4.

http://testfunda.com/LMS/Student/NewReports.aspx

10/47

Accentures overall quality score = (16 + 0 + 4 + 8)/4 = 7


Hence, option 1.
21.

3 Marks

What rating does TCS has in coding?


1)

2)

3)

4)

Solution:
TCSs rating: Design = B (Score 8), Testing = C (Score 4)
TCSs overall rating = 3
Sum of the four ratings = 3 4 = 12
Sum of ratings of Coding and Documentation = 12 8 4 = 0
TCSs scores in Coding and Documentation = 0
i.e., TCSs rating in Coding = D
Hence, option 3.

22.

3 Marks

Which of the following statement is/are true?


I. CTS gets its highest rating in testing.
II. Wipro and IBM get A rating both in documentation and coding.
III. Cognizant gets D rating for design.
1)

I and II

2)

Only II

3)

II and III

4)

Only I

Solution:
Statement I is true.
As seen in the previous question, TCSs rating in Documentation = D
IBMs rating in Design = D
IBMs score in Coding = (9 4) 0 4 16 = 16
Thus, IBMs ratings in both Documentation and Coding = A
Sum of scores of Wipros ratings in Documentation and Coding = (11 4) 4 8 = 32
Thus, Wipros scores in both Documentation and Coding = 16
Thus, statement II is also true.
Wipros score in Coding = Cognizants score in Testing = 16
Overall quality scores of both the companies are same.
Cognizants score in Design = (11 4) 8 16 16 = 4
Thus, statement III is not true.
Hence, option 1.

23.

3 Marks

There are three friends namely A, B and C and they always work in pair. They replace one by another to finish the work. They work out
in following three ways.
1. Two days after A and B started working, A was replaced by C.
2. Four days after B and C started working, B was replaced by A.
3. C and A started working and from the second day,C was replaced by B.

http://testfunda.com/LMS/Student/NewReports.aspx

11/47

They complete the work in exactly five days whichever way they follow. Find the number of days required for A, B and C respectively to
complete the work alone.
1)

8 , 10 and 12

2)

12 , 15 and 18

3)

10 , 10 and 10

4)

8 , 8 and 8

Solution:
Let the workdone by A, B, C be a, b and c units. Let the total work is W units.
Way 1: W = (2a + 2b) + (3c + 3b) = 2a + 5b + 3c (i)
Way 2: W = (4b + 4c) + (a + c) = a + 4b + 5c

(ii)

Way 3: W = (a + c) + (4a + 4b) = 5a + 4b + c

(iii)

From (ii) and (iii), a = c


From (i) and (ii), a + b = 2c
Substituting a = c, we get b = c
Thus, a = b = c
And W = 10a
Thus, each of the three working alone will require 10 days to complete the work.
Hence, option 3.

24.

3 Marks

Find unequal positive integer solution sets of ordered pairs (x, y) such that

1)

2)

3)

4)

Solution:
Let us see a generalized form of these equations.

So, for y to be an integer, (x n) should be a factor of n2.


Here, n = 21 and n2 = 212 = 32 72
So, it will have (2 + 1) (2 + 1) = 9 factors.
Hence, (x n) will take 9 different values and hence x will take 9 different values.
But there will be one value of x when x = y = 42.

http://testfunda.com/LMS/Student/NewReports.aspx

12/47

As we want only unequal values of x and y, there are 9 1 = 8 ordered pairs of (x, y).
Hence, option 1.
25.

3 Marks

If three points (p, 0), (0, q) and (2, 2) are collinear, then which of the following statements is true?
1)

2)

3)

4)

Solution:
Equation of a line in two intercepts form:

Where a and b are intercepts on x-axis and y-axis respectively.


Here, a = p and b = q, equation of the line becomes:

Since the line also passes through (2, 2),

Hence, option 4.
26.

3 Marks

Find the range of x, if

1)

x < 7 and 4 < x < 4

2)

4<x<7

3)

7 < x < 4

4)

None of these

Solution:

Let us take two cases:

Hence, |x| > 7, and hence, x (, 7) (7, )

http://testfunda.com/LMS/Student/NewReports.aspx

13/47

So, x (4, 4)
Combining both the cases, we get:
x (, 7) (4, 4) (7, )
Hence, option 4.

27.

3 Marks

1)

4093

2)

624

3)

2046

4)

1049

Solution:
2f(xy) = [f(x)]y + [f(y)]x
Replacing y by 1, we get,
2f(x) = [f(x)] + [f(1)]x
f(x) = 2x

= 21 + 22 + ... + 210

= 2046
Hence, option 3.

Group Question
Answer the following question based on the information given below.
The pie charts below depict the expense breakup for Mr. and Mrs. Puri, both of whom have independent sources of income. The 3rd pie chart
provides the division of their combined savings.
Income = Savings + Expenses

http://testfunda.com/LMS/Student/NewReports.aspx

14/47

28.

3 Marks

If the total amount in liquid savings is twice the savings of Mrs. Puri, what is the ratio of incomes of Mr. Puri and Mrs. Puri?
1)

5 : 21

2)

20 : 17

3)

21 : 5

4)

7:8

Solution:
The amount in liquid savings = 25% of the total savings
The total amount in liquid savings is twice the savings of Mrs. Puri
Savings of Mrs. Puri = 12.5% of total savings
and this represents 12% of her income.
i.e. 12% of Mrs. Puri's income = 12.5% of total savings
Mrs. Puri's income = (12.5/12) total savings
Similarly, 87.5% of total savings represents 20% of Mr. Puri's income.
Mr. Puri's income = (87.5/20) total savings
The ratio of incomes of Mr. Puri and Mrs. Puri will be (87.5/20) : (12.5/12) = (87.5 12) : (12.5 20) = 21 : 5
Hence, option 3.

29.

3 Marks

If the expenditure on Education by Mr. Puri is five times the house rent borne by Mrs. Puri, then the savings done by Mr. Puri are
approximately more than the savings of Mrs. Puri by :
1)

216%

2)

116%

3)

76%

4)

316%

Solution:
The expenditure on education by Mr. Puri = 30% of his income

http://testfunda.com/LMS/Student/NewReports.aspx

15/47

This is equal to 5 15% of the income of Mrs. Puri.


Hence, the ratio of the incomes of Mr. and Mrs. Puri = (15 5) : 30 = 15 : 6
The ratio of the savings of Mr. and Mrs. Puri = (20 15) : (12 6) = 25 : 6
Mr. Puri's savings are approximately more than Mrs. Puri's savings by (25 6)/6 316%.
Hence, option 4.
30.

3 Marks

If the savings of Mrs. Puri are equal to the total savings in Mutual funds and Bank deposits put together, then the number of areas
where the expenses of Mrs. Puri are greater than the corresponding figures for Mr. Puri is :
1)

2)

3)

4)

Solution:
45% of total savings = Savings of Mrs. Puri
Savings of Mr. Puri = 55% of total savings
The ratio of savings of Mr. Puri and Mrs. Puri will be 55 : 45
Savings of Mr. Puri is 20% of his income and savings of Mrs. Puri is 12% of her income,
The ratio of incomes of Mr. Puri and Mrs. Puri will be (55 12) : (45 20) = 11 : 15
Mrs. Puri's expenses will definitely be greater than the corresponding figures for Mr. Puri in those areas where she does not put
aside a lesser proportion of her income compared to Mr. Puri .
For example, if we consider fuel, 15% of 15x will be definitely greater than 10% of 11x.
By observation, the expenses of Mrs. Puri will be greater than the corresponding expenses of Mr. Puri for Education, House
Rent, Home Expenses and Fuel - a total of 4 areas.
Hence, option 2.

31.

3 Marks

Both, Mr. and Mrs. Puri, spend equal amounts on education. Also, the savings of Mr. Puri were Rs. 8,000. What amount did the
couple put down as liquid savings?
1)

Rs. 3,500

2)

Rs. 2,000

3)

Rs. 3,200

4)

Rs. 2,900

Solution:
Mr. and Mrs. Puri spend equal amounts on education. From the data given, Mr. Puri spends 30% of his income on education,
while Mrs. Puri spends 40% on the same.
30% of Mr. Puri's Income = 40% of Mrs. Puri's Income
Mr. Puri's Income : Mrs. Puri's Income = 40 : 30 = 4 : 3
Also, Mr. Puri saves 20% of his income, while Mrs. Puri saves 12% of her income.
Mr. Puri's Savings : Mrs. Puri's Saving = (4 20) : (3 12) = 20 : 9
Since Mr. Puri's savings were Rs. 8,000, we have,
Mrs. Puri's Savings = (9/20) Mr. Puri's Savings = (9/20) 8000 = Rs. 3,600
The amount put down in liquid savings was 25% of the total savings.
Amount in Liquid Savings = (25/100) Total Savings
= (25/100) (8000 + 3600)
= Rs. 2,900
Hence, option 4.
32.

3 Marks

There are two parallel lines, one of the lines has 9 points on it and the other line has 10 points on it. What is the number of triangles that

http://testfunda.com/LMS/Student/NewReports.aspx

16/47

can be formed using these points only (as vertices)?


1)

405

2)

360

3)

205

4)

765

Solution:
To form a triangle, three points need to be selected. There are two options, either select 2 points from the line having 9 points and 1
point from the line having 10 points or 2 points from the line having 10 points and 1 point from the line having 9 points.
Number of triangles = 10C1 9C2 + 10C2 9C1 = 765
Hence, option 4.

33.

3 Marks

If sequence S1 = 1, 11, 21, 31, ... upto 100 terms, and sequence S2 = 31, 36, 41, 46, ... upto 100 terms, The largest number common to
the sequences S1 and S2 is
1)

381

2)

471

3)

281

4)

521

Solution:
For the sequence S1, first term is 1 and common difference is 10,
t100 = 1 + (100 1)10 = 991
And for the sequence S2, first term is 31 and common difference is 5,
t100 = 31 + (100 1)5 = 526
Now, for the largest number which is common to both the sequences, we can see that sequence S1 has all the numbers ending with 1
but sequence S2 has numbers ending with either 1 or 6, hence the common number could end with 1 only. Also, the largest number of
sequence S1 is more than the largest number of sequence S2, the common number would be the largest number from sequence S2
ending with 1.
And, the largest number from sequence S2 ending with 1 is t99 = t100 5 = 526 5 = 521 which is also a number in the sequence S1
The largest number common to both sequences = 521
Hence, option 4.

34.

3 Marks

Consider the set N = {2, 3, 4, ..., 2n + 1}, where n is a positive integer larger than 10005. If O is the average of the odd integers in N
and E is the average of the even integers in N then what is the value of O E?
1)

2)

3)

n + 1/2n

4)

10006

Solution:
E = (2 + 4 + 6 + 8 + + 2n)/n
O = (3 + 5 + 7 + 9 + + (2n + 1))/n
= [(2 + 1) + (4 + 1) + (6 + 1) + (8 + 1) + + (2n + 1)]/n
= (2 + 4 + 6 + 8 + + 2n)/n + (1 + 1 + 1 + 1 + n times)/n
=E+1
OE=1
Hence, option 2.
Note: The information that 'n is a positive integer larger than 10005' does not affect the answer in any way.

http://testfunda.com/LMS/Student/NewReports.aspx

17/47

35.

3 Marks

Let f1(x) = x, f2(x) = x2, f3(x) = x3, , f9(x) = x9 and g(x) = f1(x) + f2(x) + + f9(x). What is g(p) + g(q) + + g(w), if p, q, r, s, t, u, v, w
are the roots of the equation x8 + x7 + x6 + x5 + x4 + x3 + x2 + x + 1 = 0?
1)

2)

3)

4)

Solution:
p, q, r, s, t, u, v and w are the roots of the equation x8 + x7 + x6 + x5 + x4 + x3 + x2 + x + 1 = 0
p8 + p7 + p6 + p5 + p4 + p3 + p2 + p + 1 = 0
Similarly, q8 + q7 + q6 + q5 + q4 + q3 + q2 + q + 1 = 0
.
.
.
Similarly, w8 + w7 + w6 + w5 + w4 + w3 + w2 + w + 1 = 0
Now, g(x) = f1(x) + f2(x) + + f9(x) = x + x2 + + x9
g(p) = p + p2 + + p9
g(p) = p (1 + p + + p8)
g(p) = p 0 = 0
Similarly,
g(q) = 0, g(r) = 0, g(w) = 0
g(p) + g(q) + + g(w) = (0 + 0 + + 0) = 0
Hence, option 2.

36.

3 Marks

A number; J; is represented as J = 123456789101112131415.. J has 118 digits. Find the remainder when J is divided by 8.
1)

2)

3)

4)

Solution:
Out of 118 digits, first 9 are formed by single digit numbers 1 to 9. Next 109 are formed by 2 digit numbers.
109 = 2 54 + 1
So, the next 108 digits are the numbers from 10-63 (54 = 63 9).
i.e., 116th, 117th and 118th digits are 6, 3 and 6.
Remainder when 123....636 is divided by 8 = Remainder when 636 is divided by 8 = 4
Hence, option 3.

37.

3 Marks

There are three cylinders - C1, C2 and C3. The ratio of height and radius of these cylinders are 3 : 6 : 5 and 2 : 3 : 4 respectively, in that
order of C1, C2 and C3. The value of Einstein factor, X, is directly proportional to the lateral surface area of a cylinder and is inversely
proportional to its volume. A comapany requires a cylinder with the lowest Einstein Factor. Which of the following cylinder(s) should the
company select?
1)

Only C1

2)

Only C3

3)

Both C1 and C3

4)

Both C1 and C2

http://testfunda.com/LMS/Student/NewReports.aspx

18/47

Solution:

Factor X is lowest for the cylinder with the highest radius, i.e. cylinder C3.
Hence, option 2.
38.

Each question is followed by two statements, A and B. Answer each question using the following instructions

3 Marks

Choose 1 if the question can be answered by using one of the statements alone but not by using the other statement alone.
Choose 2 if the question can be answered by using either of the statements alone.
Choose 3 if the question can be answered by using both statements together but not by either statement alone.
Choose 4 if the question cannot be answered on the basis of the two statements.
In ABC, D is a point on the side AC such that AB = AD. Find mCBD.

A. mABC + mACB = 60
B. mABC mACB = 30
1)

2)

3)

4)

Solution:
Let mACB = x and mCBD = y
Then mADB = x + y

...(Exterior angle)

Since ABD is isosceles, mABD = mADB = x + y


Consider statement A alone:
mABC + mACB = 60
x + y + y + x = 60
x + y = 30
But we cannot find the exact value of mCBD.
Statement A alone is not sufficient to answer the question.
Consider statement B alone:
mABC mACB = 30
x + y + y x = 30
2y = 30
y = 15
mCBD = 15
Thus, the question can be answered using statement B alone.
Hence, option 1.

39.

http://testfunda.com/LMS/Student/NewReports.aspx

19/47

3 Marks

Two sides of a triangle are a and b (a and b are integers); and the third side is 5. If (ab 7)2 = a2 + b2, the triangle is:
1)

Acute angled

2)

Right angled

3)

Obtuse angled

4)

More than one of the above

Solution:
(ab 7)2 = a2 + b2
(ab 6)2 2ab + 13 = a2 + b2
(ab 6)2 + 13 = (a+ b)2
So, (a + b)2 (ab 6)2 = 13
(a + b ab + 6) (a + b + ab 6) = 13
(a + b ab + 6) (a + b + ab 6) =

13 .

Negative values can be ignored as they will give negative values of a and b which is not possible.
Thus, a + b + ab 6 = 13 and a + b ab + 6 = 1 (1)
Or, a + b + ab 6 = 1 and a + b ab + 6 = 13 (2)
Solving (1),
a = 4 and b = 3
Or a = 3 and b = 4
If 3, 4, 5 are the sides of the triangle then the triangle is right angled triangle.
Solving (2),
a = 7 and b = 0, which is not a possible.
Hence, option 2.
40.

3 Marks

On walking a certain distance at 80 m/min, I was 2 minutes late. However, after running at twice that speed, I was 3 minutes before time.
The corresponding distance and the scheduled time was:
1)

1000 m, 8 min

2)

800 m, 8 min

3)

1000 m, 10 min

4)

None of these

Solution:
Let the distance and scheduled time be d and t respectively.
d = 80(t + 2) = 160(t 3)
On solving these equations, we get,
t = 8 minutes and d = 800 meters
Hence, option 2.
41.

3 Marks

Two numbers p and q are chosen randomly out of the set of the first six prime numbers without repetition. What is the probability that the
equation x2 px + q = 0 has two complex roots?
1)

13/15

2)

11/30

3)

2/5

4)

13/30

Solution:
We know that a quadratic equation will have two complex roots if and only if its discriminant is negative.

http://testfunda.com/LMS/Student/NewReports.aspx

20/47

In this case, the condition translates into p2 4q < 0 or p2 < 4q.


Now, the first six prime numbers are 2, 3, 5, 7, 11 and 13.
Now, we count the number of favourable cases.
If p = 2, then q = 3, 5, 7, 11 and 13 (5 sets).
If p = 3, then q = 5, 7, 11 and 13 (4 sets).
If p = 5, then q = 7, 11 and 13 (3 sets).
If p = 7, the condition is met only for q = 13.
The total number of sets of (p, q) = 5 + 4 + 3 + 1 = 13
The total number of possible (p, q) sets = 6P2 = 30
The probability of getting complex roots for the given equation = 13/30
Hence, option 4.
Group Question
Answer the following question based on the information given below.
The table below provides production data related to five crops cultivated in Maharashtra. Here, the columns represent the following items:
A = Production of crops for the year 2007 (in MT).
B = Percentage increase in the production of crops in the year 2007 as compared to the respective production in the year 2006.
C = Target production for the year 2008 (in MT).
[Note: MT represents Metric Ton]

42.

3 Marks

In 2006, what was the total production of the crops that were the top three contributors to the crop production in 2007?
1)

135 MT

2)

132 MT

3)

158 MT

4)

160 MT

Solution:
Food grains, sugarcane and cotton were the top three contributors to the crop production in 2007.
Their respective production in 2006 is as follows:
Food grains : 160.4/1.25 = 128.32 MT
Sugarcane : 22.5/1.35 = 16.67 MT
Cotton : 18.9/1.32 = 14.32 MT
So, the total crop production of these three crops in 2006 = 128.32 + 16.67 + 14.32 = 159.31 MT.
The option closest to this is 160 MT (option 4).
Hence, option 4.
43.

3 Marks

What is the percentage change in the total production of all the five crops in the Maharashtra state from year 2006 to year 2007?
1)

18%

2)

22%

3)

32%

http://testfunda.com/LMS/Student/NewReports.aspx

21/47

4)

25%

Solution:
Consider the solution to the first question.
The total production of all the five crops in the year 2007 = 160.4 + 16.2 + 14.8 + 22.5 + 18.9 = 232.8 MT
Production of food grains, sugarcane and cotton in 2006 is 128.32, 16.67 and 14.32 MT respectively.
Production of Oil Seeds in 2006 = 16.2/1.18 = 13.73 MT
Production of Jute in 2006 = 14.8/1.16 = 12.76 MT
Total production of all five crops in 2006 = 128.32 + 16.67 + 14.32 + 13.73 + 12.76 = 185.8 MT
Required percentage change = (232.8 - 185.8)/185.8 100 25.3%.
Hence, option 4.
44.

If the target production for the year 2008 is achieved for all the five crops in Maharashtra state, what is the maximum percentage
change in the production of a crop from year 2007 to year 2008?

3 Marks

1)

9.8%

2)

22.22%

3)

61.4%

4)

130.2%

Solution:
The percentage change for each crop is:
Food Grains : (162.7 160.4)/160.4 100 = 1.43%
Oil Seeds : (18.9 16.2)/16.2 100 = 16.67%
Jute : (12.3 14.8)/14.8 100 = 16.89%
Sugarcane : (20.5 22.5)/22.5 100 = 8.88%
Cotton : (14.7 18.9)/18.9 100 = 22.22%
Thus, the maximum percentage change is for Cotton.
Hence, option 2.
Note: Since we are interested only in the percentage change, percentage increase or decrease does not make a difference. We
consider only the highest magnitude.

45.

3 Marks

Four buffaloes are tied at four corners of a square field of side 14 m in such a way that any of them can just reach only two other
buffaloes. Calculate the area that remains ungrazed.
1)

154 m2

2)

42 m2

3)

38.5 m2

4)

10.5 m2

Solution:

http://testfunda.com/LMS/Student/NewReports.aspx

22/47

The buffaloes are able to graze over a circular sector of the field centered at the corner to which they are tied.
Therefore, the ungrazed portion of the field looks like the shaded portion in the figure.
The area of the shaded portion = Area of the square Area of the four quarter circles
= 142 (14/2)2
= 196 (22/7) 7 7
= 42 m2
Hence, option 2.
46.

3 Marks

If x3 x 1 = 0, then (x 1) is equal to
1)

2)

3)

4)

Solution:
Consider the equation, x3 x 1 = 0,
x3 = x + 1

...(i)

On multiplying the equation by x and x2 to get,


x4 = x2 + x

(ii)

x5 = x3 + x2

(iii)

x5 = x3 + x2 = (x + 1) + x2

[Using (i)]

= (x + x2) + 1
= x4 + 1

[Using (ii)]

x5 x4 = 1

Hence, option 4.

47.

3 Marks

If log x, log xA, log xB and log xC are in geometric progression with common ratio 2, what is the value of (A + B + C)?
1)

2)

3)

12

4)

14

Solution:
log xy = y log x

http://testfunda.com/LMS/Student/NewReports.aspx

23/47

We can rewrite the four terms of GP as log x, A log x, B log x and C log x.
Since the common ratio of the GP is 2, therefore, A log x = 2 log x, therefore, A = 2
Similarly, we have B log x = 2A log x, therefore, B = 2A = 4.
Similarly, we get, C = 8
A + B + C = 2 + 4 + 8 = 14
Hence, option 4.
48.

3 Marks

x < y < z and x, y, z are in geometric progression. If x20 = yt = z30, what is the value of t?
1)

22

2)

23

3)

24

4)

25

Solution:
Let r be the common ratio.
y = xr
and z = xr2
x20 = (xr)t = (xr2)30

(i)

x20 = x30r60
r60 = x10
r = x(1/6)
x20 = (xr)t = xt x(t/6)

from (i)

t = 24
Hence, option 3.

49.

3 Marks

Each question is followed by two statements A and B. Answer each question using the following instructions.
Mark (1) if the question can be answered by using statement A alone but not by using statement B alone.
Mark (2) if the question can be answered by using statement B alone but not by using statement A alone.
Mark (3) if the question can be answered by using both the statements together but not by using either of the statements alone.
Mark (4) if the question cannot be answered on the basis of the two statements.
Each student in a college plays Cricket or Football. If 5% of the students who play Football also play Cricket, then is it true that the
number of students who play Cricket is more than the number of students who play Football?
A. The number of students who play both Cricket and Football is 11.
B. 12% of the students who play Cricket also play Football.
1)

2)

3)

4)

Solution:
Let the number of students who play Cricket be C and those who play Football be F.
The number of students who play both Cricket and Football = 5%(F)

(i)

We have to find the greater of F and C.


Consider statement A alone:
11 students play both Cricket and Football.
From (i), we have 5%(F) = 11
F = 220

(ii)

But C cannot be found.

http://testfunda.com/LMS/Student/NewReports.aspx

24/47

So statement A alone is not sufficient to answer the question.


Consider statement B alone:
12% of the students who play Cricket also play Football.
The number of students who play both Cricket and Football = 12%(C)

(iii)

From (i) and (iii), we have


5%(F) = 12%(C)

Thus, we have C < F.


So statement B alone is sufficient to answer the question.
Hence, option 2.

50.

3 Marks

Each question is followed by two statements A and B. Answer each question using the following instructions.
Mark (1) if the question can be answered by using statement A alone but not by using statement B alone.
Mark (2) if the question can be answered by using statement B alone but not by using statement A alone.
Mark (3) if the question can be answered by using both the statements together but not by using either of the statements alone.
Mark (4) if the question cannot be answered on the basis of the two statements.
A microwave and a dishwasher were sold at a profit of 10% and 20% respectively. What is the overall profit percentage?
A. The cost of the dishwasher is 50% more than that of the microwave.
B. The cost of the dishwasher is Rs. 7000 more than that of the microwave.
1)

2)

3)

4)

Solution:
Consider Statement A alone:
The cost of the dishwasher is 50% more than that of the microwave.
So, if the cost price of the microwave is 2x, then that of the dishwasher will be 3x.
Total Cost Price = 2x + 3x = 5x

(i)

Now, Selling Price of the dishwasher = 1.2 3x = 3.6x


Selling Price of the microwave = 1.1 2x = 2.2x
Total Selling Price = 3.6x + 2.2x = 5.8x

(ii)

From (i) and (ii), we get,


Total profit percentage = (5.8x 5x)/5x 100 = 0.8x/5x 100 = 16%
Hence statement A alone is sufficient.
Consider Statement B alone:
The cost of the dishwasher is Rs. 7000 more than that of the microwave.
If the cost price of the microwave is Rs. x, then that of the dishwasher will be Rs. (x + 7000).
Total Cost Price = 2x + 7000

(iii)

Now, selling price of the dishwasher = 1.2 (x + 7000) = 1.2x + 8400


Selling price of the microwave = 1.1x
Total selling price = 2.3x + 8400

(iv)

From (iii) and (iv), we get,


Total profit percentage = ((2.3x + 8400) (2x + 7000))/(2x + 7000) 100
= (0.3x + 1400)/(2x + 7000) 100
Since we do not know the value of x, the profit percentage cannot be found.

http://testfunda.com/LMS/Student/NewReports.aspx

25/47

Hence statement B alone is not sufficient.


Hence, option 1.

Section II
51.

3 Marks

The following question has a paragraph from which the last part of the last sentence has been deleted. From the given options, choose
the one that completes the paragraph in the most appropriate way.
But at moments such as these something has to be done. The man who doesn't like it, let him dislike it ever so much, cannot check his
horse and simply ride back to the hunting stables. He understands that were he to do that, he must throw up his cap at once and resign.
Nor can he trot easily along the roads with the fat old country gentleman who is out on his rough cob, and who, looking up to the wind
and remembering the position of adjacent coverts, will give a good guess as to the direction in which the field will move. No; he must
make an effort. The time of his penance has come, and the penance must be borne. There is a spark of pluck about him, though
unfortunately he has brought it to bear in a wrong direction. The blood still runs at his heart, and he resolves that he will
ride,__________________________.
1)

if only he could tell which way.

2)

as he must to answer his critics.

3)

as easily as the fat old country gentleman trotting along the roads and waiting for something to show him the way.

4)

as the penance of his past.

Solution:
The paragraph explains the journey of man and shows his state of confusion. It illustrates the fact that he must face penance for his
deeds. It asserts the point that he has to make an effort and not give away to easy options. This points to option 1 as the correct answer.
The paragraph does not criticize the rider, it just describes his state of mind and his situation and therefore option 2 can be eliminated.
Option 3 is incorrect as the paragraph states that person in discussion will not ride the way in which the old fat gentleman does.
It has been illustrated that the rider must bear his penance. Though option 4 seems tempting, only option 1 highlights the confused state
of mind of the rider and resonates with the idea portrayed in the paragraph initially. Hence, it makes for a more apt conclusion for the
paragraph.
Hence, the correct answer is option 1.
52.

3 Marks

Fill in the blanks with the most appropriate pair of words from the given options.
The Indian traditional habit of marrying within ones caste or community leads to genetic mutations, thus explaining why certain diseases
are ______ only in a particular pocket of the _______ in India.
1)

found, community

2)

contracted, people

3)

prevalent, population

4)

diagnosed, majority

Solution:
Since according to the context, genetic mutations take place within the community through marriages that happen only within the
community, certain diseases are inherent and passed on from one generation of the community to the other. The context thus, tells us
that these diseases are generally found or prevalent only in that particular pocket of the people belonging to that community.
Pocket of eliminates the word community as it will further restrict the number of people within the community.
Pocket of people will refer to a community, and will be more correctly put as a pocket of population. Therefore, we can eliminate
options 1, 2 and 4.
Prevalent is better than found and communicates the occurrence and continuity of diseases in a more appropriate manner.
Hence, the correct answer is option 3.
53.

3 Marks

A base word has been used in the options given below. Choose the option in which the usage of the word stands out as ODD in
comparison to the other options.
Retreat
1)

The opposition army had to retreat after incurring huge losses in the first few days of the battle.

2)

The commander of the army moved to a retreat, far away from the media glare.

3)

The governments of two countries retreated from their earlier tough stand on transnational corporate taxes.

4)

Coming under fire from the activist group, the companies decided to retreat their statements on the issue of the environment.

Solution:
The meaning of the given word in options 1, 3, and 4 is to withdraw, move backwards where as the meaning in option 2 refers to a
haven or a refuge.
Hence, the correct answer is option 2.
Group Question

http://testfunda.com/LMS/Student/NewReports.aspx

26/47

A passage is followed by questions pertaining to the passage. Read the passage and answer the questions. Choose the most appropriate answer.
Nietzsche is considered unique among philosophers by some scholars for what is widely regarded as the power and effectiveness of his
rhetorical style particularly as manifested in Zarathustra. The indigestible "heaviness" long associated with German-language philosophy is
eschewed, with puns and paradoxes abounding, and aphoristic brevity characteristic of parable and even poetry. The end result is a manner
of writing which, being "pitched half-way between metaphor and literal statement", is "something quite extraordinary".
Noteworthy for its format, the book comprises a philosophical work of fiction whose style often light heartedly imitates that of the New
Testament and of the Platonic dialogues, at times resembling pre-Socratic works in tone and in its use of natural phenomena as rhetorical and
explanatory devices. It also features frequent references to the Western literary and philosophical traditions, implicitly offering an interpretation
of these traditions and of their problems. Nietzsche achieves all of this through the character of Zarathustra (referring to the traditional prophet
of Zoroastrianism), who makes speeches on philosophic topics as he moves along a loose plotline marking his development and the reception
of his ideas. One can view this characteristic (following the genre of the bildungsroman) as an inline commentary on Zarathustra's (and
Nietzsche's) philosophy. All this, along with the book's ambiguity and paradoxical nature, has helped its eventual enthusiastic reception by the
reading public, but has frustrated academic attempts at analysis (as Nietzsche may have intended). Thus Spoke Zarathustra remained
unpopular as a topic for scholars (especially those in the Anglo-American analytic tradition) until the second half of the twentieth century
brought widespread interest in Nietzsche and his unconventional style that does not distinguish between philosophy and literature. It offers
formulations of eternal recurrence, and Nietzsche for the first time speaks of the bermensch: themes that would dominate his books from this
point onwards.
A vulnerability of Nietzsche's style is that his nuances and shades of meaning are very easily lost - and all too easily gained - in translation.
The bermensch is particularly problematic: the equivalent "Superman" found in dictionaries and in the translations by Thomas Common and
R.J. Hollingdale may create an unfortunate association with the heroic comic-character "Superman", while simultaneously detracting from
Nietzsche's repeated play on "ber" as well as losing the gender-neutrality of the German.
The "bermensch" is the being that overcomes the "great nausea" associated with nihilism; that overcomes that most "abysmal" realization of
the eternal return. He is the being that "sails over morality", and that dances over gravity (the "spirit of gravity" is Zarathustra's devil and
archenemy). He is a "harvester" and a "celebrant" who endlessly affirms his existence, thereby becoming the transfigurer of his
consciousness and life, aesthetically. He is initially a destructive force, excising and annihilating the insidious "truths" of the herd, and
consequently reclaiming the chaos from which pure creativity is born. It is this creative force exemplified by the bermensch that justifies
suffering without displacing it in some "afterworld".
54.

3 Marks

Which of the following is true about Nietzsche?


1)

Nietzsches work appeared as a divergence from the traditional German philosophical writing traditions.

2)

Nietzsches ideas have survived mainly because of the effective translation and literary review by other scholars.

3)

Nietzsches work distinguishes literature from philosophy.

4)

Nietzsche was a prolific writer, having authored a series of books, articles, magazines etc on a range of subjects from
morality to humanism.

Solution:
The passage states the following distinction between Nietzche's work and traditional German philosophical writing, The
indigestible "heaviness" long associated with German-language philosophy is eschewed..... This vindicates option 1.
Option 2 is negated as the second last paragraph mentions that Nietzsches work is vulnerable to the inaccuracies of translation
from German to other languages.
Option 3 is incorrect as can be seen from the line Nietzsche and his unconventional style that does not distinguish between
philosophy and literature..
Option 4 is negated as the passage does not mention the volume of Nietzsches work, it merely describes his writing style and is
largely focused on his book Thus spoke Zarathustra.
Hence, the correct answer is option 1.
55.

3 Marks

Which of the following cannot be said about bermensch?


1)

It relegates suffering to the after world to unleash the forces of creativity.

2)

It deals with internal metamorphosis.

3)

It was an underlying theme for much of Nietzsches work.

4)

It represents struggle against conventional norms and morality.

Solution:
According to the passage, bermensch ...is a "harvester" and a "celebrant" who endlessly affirms his existence, thereby
becoming the transfigurer of his consciousness and life, aesthetically.. Thus, option 2 holds true for bermensch.
Option 3 is validated by Nietzsche for the first time speaks of the bermensch: themes that would dominate his books from this
point onwards..
Option 4 also holds true for bermensch as can be vindicated by, He is the being that "sails over morality"...He is initially a
destructive force, excising and annihilating the insidious "truths" of the herd....
Option 1 is contrary to the information in the passage, which clearly mentions that suffering is not displaced to any after world.
Hence, the correct answer is option 1.
56.

3 Marks

The book Thus spoke Zarathustra is all of the following EXCEPT


1)

the initial effort towards Nietzsches concept of bermensch.

2)

a commentary on Nietzsches philosophy reflected in the details of Zarathustras ideas.

3)

perhaps intentionally made ambiguous by Nietzsche to vex academic interpretation.

http://testfunda.com/LMS/Student/NewReports.aspx

27/47

4)

None of the above

Solution:
A detailed reading of the second paragraph can help arrive at the correct answer.
Option 1 is vindicated by It offers formulations of eternal recurrence, and Nietzsche for the first time speaks of the bermensch:
themes that would dominate his books from this point onwards..
Option 2 is ratified by One can view this characteristic (following the genre of the bildungsroman) as an inline commentary on
Zarathustra's (and Nietzsche's) philosophy.
Option 3 is validated by All this, along with the book's ambiguity and paradoxical nature, has helped its eventual enthusiastic
reception by the reading public, but has frustrated academic attempts at analysis (as Nietzsche may have intended)..
Hence, the correct answer is option 4.
57.

bermensch as a concept in the original German is:

3 Marks

I. Morality neutral
II. Gender neutral
III. Often contradictory in form and nature

1)

Both I and II

2)

Both II and III

3)

All of I, II and III

4)

None of these

Solution:
bermensch is said to rise above the constraints of morality and therefore, statement I is correct.
Statement II is correct as the term gets a gender connotation only when translated. It appears to be gender neutral in the German
language.
Statement III is correct as bermensch appears to be a destructive force first and then a creative force.
Hence, the correct answer is option 3.
58.

3 Marks

Four sentences numbered I, II, III and IV are given in random order. Three of these sentences belong to the same paragraph, and can be
arranged logically to form a coherent paragraph. One of the sentences does not fit into this paragraph. Choose the ODD sentence for
your answer.
I. ICT could help in combating climate change, by encouraging, adopting green practices and developing of more energy efficient
devices, applications and networks.
II. Information and communications technology can play a major role in combating climate change by encouraging adoption of green
practices.
III. The fabled report is just a step forward; with a rapid growth of ICT sector makes it imperative that in future, it needs to run in an
energy efficient manner.
IV. ICT can be a key enabler which can promote growth by a low carbon economy.
1)

2)

II

3)

III

4)

IV

Solution:
Statement II puts forth the role that Information and communications technology... will play in combating climate change by its
encouragement of green practices. The other statements in the sequence also address the issue of green practices. Eliminate option 2.
Statements II, I and IV make a logical sequence as statement I further develops on the role that ICT can play in combating climate
change as put forth in statement II. Statement IV brings in the aspect of ...a low carbon economy, which goes with green practices.
Eliminate options 1 and 4.
The report mentioned in statement III is vague and does not relate to anything mentioned in the other sentences.
Hence, the correct answer is option 3.
59.

Identify the grammatically and logically incorrect sentences.

3 Marks

A. Playing the lottery means trying to make a connection with divine providence and rejecting the narcissistic idea that success is
due solely to our own efforts - in short, accepting grace over merit.
B. It also means accepting a fundamental inequality in which were all children of God, though some of us are more favoured than
others.
C. Good luck is a blessing, a sign from heaven; even bad luck acquires a deeper and more bearable meaning when its understood
as inscrutable divine will.
D. Good luck is a less offensive privilege than merit.
E. When accession on the throne is determined by birth order, the second- and third-born are considered unlucky, not unworthy.

1)

A and E

2)

B and D

http://testfunda.com/LMS/Student/NewReports.aspx

28/47

3)

A and C

4)

B and E

Solution:
Statement B is logically incoherent since the phrase ... were all children of God ... implies a sense of equality and not inequality.
Moreover, the usage of though indicates that inequality ought to be an exception and not something fundamental. This eliminates
options 1 and 3.
Statement E contains a prepositional error; the usage of on with accession is incorrect. The correct usage would be accession to the
throne. This eliminates option 2.
Statements A, C and D are grammatically and logically coherent.
Hence, the correct answer is option 4.
60.

3 Marks

The following question has a paragraph from which the last sentence has been deleted. From the given options, choose the one that
completes the paragraph in the most appropriate way.
I dont do any reviews or critiques; I dont want to be involved in that. I think that my job is to observe people and the world, and not to
judge them. I always hope to position myself away from so-called conclusions. ______.
1)

This way, when I start to write, I dont have any plan at all.

2)

We need critics in this world, for sure, but its just not my job.

3)

I would like to leave everything wide open to all the possibilities in the world.

4)

I prefer translating to criticism, because you are hardly required to judge anything when you translate.

Solution:
The paragraph describes the authors perspective on observing people. The sentence that would suitably complete it, must articulate
these views in the best possible manner, thereby putting forth a reason why the author may feel so. Since the author mentions that
he/she wishes to observe and stay away from conclusions and judgments, we can infer that he/she wishes to leave room for possibilities.
This vindicates option 3 as correct.
Option 1 seems rather far-fetched, since the author does not put forth his/her plans in the paragraph.
Option 2 fails to take the last line into consideration and hence, loses out to option 3.
Option 4 moves beyond the scope of the paragraph which limits itself to judgements and conclusions about people.
Hence, the correct answer is option 3.
61.

3 Marks

The question below consists of a set of labelled sentences. These sentences, when properly sequenced, form a coherent paragraph.
Choose the most logical order of sentences from the options.
I. And these almost always serve the didactic purpose of presenting science as a noble, brave and objective quest for truth in the
face of ideology and superstition, whether that is Galileo versus the pope or Einstein versus the Nazis.
II. Its surely the attraction of heroic vision that explains why scientists play fast and loose with history.
III. Scientists do seem to have an unusual susceptibility for bowdlerised narrative, pantheons and idols.
IV. In these stories, great scientists shake off the shackles, while dogma and prejudice capitulate to unreason.
V. History almost inevitably gets simplified in the popular retelling, and all trades and institutions invent their own myths.
1)

V, III, IV, I, II

2)

II, V, III, I, IV

3)

III, IV, I, V, II

4)

II, III, V, IV, I

Solution:
The sequence pertains to the susceptibility of scientists towards ...bowdlerised narrative, pantheons and idols., what happens to history
with the passage of time, how it impacts the narrative in science and what eventually happens to scientists on account of this
susceptibility.
The options provide three choices as sequence starters, statements V, II and III.
While option 1 beginning with statement V does seem to flow, it is faulty in placing statement I after statement IV. If through the
narratives, great scientists shake off the shackles, and dogma and prejudice concede to unreason, then to state that the didactic purpose
of science is an objective quest for the truth (as suggested in statement I) does not make sense. There is no basis on which it can be
described as noble or brave if statement IV is placed before I. Statements II, V and III serve better in preceding statement I as they focus
on the aspect of myths and idolization in science, which statement I elaborates upon. Statement IV serves better to consolidate the
thought conveyed through the sequence. Options 3 and 4 can also be eliminated on the same basis.
Both statements II and V address the interaction of scientists with history. Statement III discusses what makes scientists vulnerable and
statements I and IV in that order further expound the line of thought mentioned in statement III. Thus, the most logical order of the
sequence is II, V, III, I, IV.
Hence, the correct answer is option 2.
Group Question
Answer the following question based on the information given below.
Eight paper slips numbered from 1 to 8, have a question each from either LR-DI or QA or VCA area. The total number of questions from any
of the area in all the slips put together is at least one and not more than three. These slips were then picked up by eight students: A, B, C, D,
E, F, G and H, who are seated linearly from left to right in that order.

http://testfunda.com/LMS/Student/NewReports.aspx

29/47

Also,
1.
2.
3.
4.

No two questions from same area belong to any two consecutively numbered slips.
No two consecutively seated students picked up a question from either VCA or LR-DI area.
Three consecutively seated students picked up questions from QA area.
A and G picked up question from LR-DI and VCA area respectively.

62.

3 Marks

From which area did D pick up the question?


1)

LR-DI

2)

QA

3)

VCA

4)

Cannot be determined

Solution:
A: LR-DI and G: VCA
From condition 2,
B cannot pick LR-DI and F and H cannot pick VCA.
From condition 3, three consecutively seated people picking QA can be:
Case 1: B, C, D
Case 2: C, D, E
Case 3: D, E, F
Thus H definitely picks up LR-DI.
In all the three cases, D is common.
Thus D definitely picked up the question on QA.
Hence, option 2.

63.

3 Marks

If A, C and G picked up paper slips numbered 5, 4 and 3 respectively, then which of the following is false?
1)

B picked up paper slip 2.

2)

D and E picked up slips 6 and 7 respectively.

3)

B and E picked up paper slips 6 and 8 respectively.

4)

F and H picked up slips numbered 6 and 7 respectively.

Solution:
A: (LR-DI, 5)
G: (VCA, 3)
From condition 1, slip number 4 must contain a question on QA.
Thus C: (QA, 4)
As C and D have question on QA, from condition 3, either B or E must have a question on QA.
Thus F definitely does not have QA which implies F has LR-DI.
Since both H and F have a question on LR-DI, number on the slips that they have cannot be consecutive.
Thus, option 4 is definitely false.
Hence, option 4.
64.

3 Marks

If E and C picked up slips 1 and 2 respectively, then each of the following statements could be true, except...
1)

D picked up slip 3.

2)

F and H picked up slips 3 and 4 respectively.

3)

A and H picked up slips numbered 5 and 6 respectively.

4)

Both 1 and 2

http://testfunda.com/LMS/Student/NewReports.aspx

30/47

Solution:
Option 1 is possible as shown below:

Option 2 is possible as shown below:

Option 3: A and H picked up question from LR-DI area. So these slips cannot have consecutive numbering.
Hence, option 3.

65.

3 Marks

The following question has a paragraph from which the last sentence has been deleted. From the given options, choose the one that
completes the paragraph in the most appropriate way.
At the end of a meeting in China, it is customary to thank the participants for contributing their valuable time. Punctuality on arrival is also
considered importantmore so than in many other Asian countries. Indeed, when meetings are scheduled between two people, it is not
unusual for a Chinese to arrive 15 to 30 minutes early in order to finish the business before the time appointed for its discussion, so not
stealing any of the other persons time! It is also considered polite in China to announce, 10 or 15 minutes after a meeting has begun,
that one will soon have to be going. _______.
1)

This is to ensure that both parties involved in the discussion respect time constraints.

2)

The Chinese penchant for humility demands that the other persons time be seen as precious.

3)

Again, the worthy aim involved is to economize on their use of your time.

4)

The Chinese, like most Asians, walk around the pool in order to make well-considered decisions, but they have a keen sense
of the value of time.

Solution:
The paragraph describes the Chinese perspective on the utilization of time. Chinese customs emphasize on punctuality and respecting
the time allotted to one by the other party. The last line of the paragraph mentions how it is considered polite to announce that one will
have to be going soon. The line that completes the paragraph must link this custom to the sense of humility that the Chinese have for
others time which is implied by the rest of the paragraph. Option 3 alone explains the action further - the custom is a way of reassuring
the other party that one will not take up more time than necessary.
Option 1 calls into question the expectations of the Chinese from the other party which have not been detailed upon in the passage.
Option 2 is redundant and does not delve further into the custom mentioned in the last line.
Option 4 introduces the aspect of decision-making which has not been touched upon in the paragraph.
Hence, the correct answer is option 3.
Group Question
The passage given below is followed by a set of questions. Choose the most appropriate answer to each question.
My invisibility isnt exactly a matter of a bio-chemical accident to my epidermis. That invisibility to which I refer occurs because of a peculiar
disposition of the eyes of those with whom I come in contact. A matter of the construction of their inner eyes, those eyes with which they look
through their physical eyes upon reality. I am not complaining, nor am I protesting either. It is sometimes advantageous to be unseen,
although it is most often rather wearing on the nerves. Then too, you're constantly being bumped against by those of poor vision. Or again,
you often doubt if you really exist. You wonder whether you aren't simply a phantom in other people's minds. Say, a figure in a nightmare
which the sleeper tries with all his strength to destroy. It's when you feel like this that, out of resentment, you begin to bump people back. And,
let me confess, you feel that way most of the time. You ache with the need to convince yourself that you do exist in the real world, that you're
a part of all the sound and anguish, and you strike out with your fists, you curse and you swear to make them recognize you. And, alas, it's
seldom successful. Most of the time, I remember that I am invisible and walk softly so as not to awaken the sleeping ones. Sometimes it is
best not to awaken them; there are few things in the world as dangerous as sleepwalkers. I learned in time though that it is possible to carry
on a fight against them without their realizing it.
Our whole world sounds like a contradiction, but that is just how it moves. Not like an arrow, but a boomerang. Beware of those who speak of
the spiral of history; they are preparing a boomerang. Keep a steel helmet handy. I have been boomeranged across my head so much that I
now can see the darkness of lightness. And I love light. Perhaps you'll think it strange that an invisible man should need light, desire light, love
light. But maybe it is exactly because I am invisible. Light confirms my reality, gives birth to my form. A beautiful girl once told me of a
recurring nightmare in which she lay in the center of a large dark room and felt her face expand until it filled the whole room, becoming a
formless mass while her eyes ran in bilious jelly up the chimney. And so it is with me. Without light I am not only invisible, but formless as well;
and to be unaware of one's form is to live a death. I myself, after existing some twenty years, did not become alive until I discovered my
invisibility.
66.

3 Marks

Which of the following cannot be attributed to the narrator of this passage?


1)

A feeling of incongruity about his/her own existence.

2)

Confusion and doubt about the nature of his/her existence.

3)

Cynicism about positive change in his/her circumstances.

http://testfunda.com/LMS/Student/NewReports.aspx

31/47

4)

Acceptance of his/her rejection by mainstream society.

Solution:
The narrator elaborates on the anguish and disbelief that he/she feels towards his/her existence ...you often doubt if you really
exist. You wonder whether you aren't simply a phantom in other people's minds. Say, a figure in a nightmare which the sleeper
tries with all his strength to destroy.. This helps us infer that the narrator feels out of place wherever he/she is, which is supported
by You ache with the need to convince yourself that you do exist in the real world, that you're a part of all the sound and anguish
.... This supports options 1 and 2.
Option 4 can be inferred from Most of the time, I remember that I am invisible and walk softly so as not to awaken the sleeping
ones.. This implies that the narrator has come to terms with his/her circumstances.
Though the passage mentions ... you strike out with your fists, you curse and you swear to make them recognize you. And, alas,
it's seldom successful., it implies that the narrator is cynical whether his/her expression of anguish would result in an acceptance
of his/her existence rather than being cynical towards positive change. Additionally in the second paragraph, the author highlights
the importance of light and how it has impacted his/her life. According to the author acceptance of his/her situation helped him/her
in feeling alive. Thus, we can infer that the author is inclined towards positive change. This vindicates option 3 as correct.
Hence, the correct answer is option 3.
67.

Which of the following cannot be concluded on the basis of the passage?

3 Marks

1)

It could be dangerous for the narrator to be a part of society.

2)

The narrator seeks a deeper understanding of his/her being.

3)

The narrators invisibility is perceived negatively by society.

4)

The narrators awareness of the denial of his/her existence is what reinforces it.

Solution:
Option 1 can be inferred from Then too, you're constantly being bumped against by those of poor vision. Or again, you often
doubt if you really exist. You wonder whether you aren't simply a phantom in other people's minds. Say, a figure in a nightmare
which the sleeper tries with all his strength to destroy. and Sometimes it is best not to awaken them; there are few things in the
world as dangerous as sleepwalkers.. This implies that the narrators inclusion in mainstream society endangers him/her.
The passage uses metaphors to detail upon the conditions of the narrator. The denial of his/her existence by the society has been
referred to as a form of invisibility and the light refers to knowledge and understanding. According to the passage, I have been
boomeranged across my head so much that I now can see the darkness of lightness. And I love light. Perhaps you'll think it
strange that an invisible man should need light, desire light, love light.. This eliminates option 2.
Option 4 is supported by Without light I am not only invisible, but formless as well; and to be unaware of one's form is to live a
death. I myself, after existing some twenty years, did not become alive until I discovered my invisibility..
We cannot verify the validity of option 3 since the passage pertains to how the society perceives the narrators existence
negatively and does not deliberate upon society's perception of its own denial of the narrator's existence.
Hence, the correct answer is option 3.
68.

Which of the following accurately describes the combination of writing styles used in the passage?

3 Marks

I.
II.
III.
IV.
V.

Reflective
Narrative
Descriptive
Metaphysical
Emotive

1)

I and II only

2)

II, IV and V

3)

III and IV only

4)

All of the above

Solution:
The passage puts forth the narrators anguish and elucidates how the narrator perceives his/her condition to be. Thus, the
passage is certainly reflective meaning concerned with mediation or deliberation. This eliminates options 2 and 3.
The passage is put forth as an account of the authors thoughts and experiences, each one being elucidated upon thoroughly.
This helps us infer that the passage uses a narrative style as well as a descriptive one. Eliminate option 1.
The narrator uses several metaphors to describe his/her existential angst and is not literal in his/her narration. He/she touches
upon several philosophical concepts in putting forth how the denial of existence can affect a human being. Thus, the passage is
certainly metaphysical in its narration.
The passage is centered on how the denial of existence affects the narrator and he/she gives voice to his/her feelings throughout
the passage. This establishes the emotive nature of the authors writing and vindicates option 4 as correct.
Hence, the correct answer is option 4.

69.

3 Marks

A base word has been used in the options given below. Choose the option in which the usage of the word is incorrect or inappropriate.
DUE
1)

My wife is due in three weeks.

2)

He deserves to be given his due.

3)

The island lies due south of the Lighthouse.

http://testfunda.com/LMS/Student/NewReports.aspx

32/47

4)

I must pay my dues and admit that my diabolical boss gets things done.

Solution:
In option 1 due is used to mean expected to give birth. Its usage is correct.
In option 2 due is used as a noun to mean something that rightfully belongs to one, e.g., give him his due.
In option 3 due is used as an adverb to mean exactly or directly. Its usage is correct.
The usage of the adjective diabolical meaning having the qualities of a devil with the word boss indicates that the person harbours
negative feelings towards his/her boss. Therefore, the correct idiom to use in option 4 would be give the devil his due meaning 'to admit
that someone you do not admire or like does have some good qualities'. Additionally pay one's dues means to earn respect, a position,
or a right by hard work, sacrifice, or experience.
Hence, the correct answer is option 4.
70.

3 Marks

The following question has a paragraph from which the last sentence has been deleted. From the given options, choose the one that
completes the paragraph in the most appropriate way.
The launch of trilateral strategic consultations among the United States, India, and Japan, and their decision to hold joint naval exercises
signals efforts to form an entente among the Asia-Pacific regions three leading democracies. At a time when Asia is in transition and
troubled by growing security challenges, the US, India, and Japan are seeking to build a broader strategic understanding to advance
their shared interests. Their effort calls to mind the pre-World War I Franco-British-Russian Triple Entente to meet the threat posed by
the rapid rise of an increasingly assertive Germany. ______________________ .
1)

This entente could serve as an important strategic instrument to deter Chinas rising power from sliding into arrogance.

2)

Indeed, the intention of the three democratic powers is to create an entente cordiale without transforming it into a formal
military alliance.

3)

The new strategy supports India as a regional economic anchor and provider of security in the broader Indian Ocean region.

4)

This time, the impetus has been provided by Chinas increasingly muscular foreign policy.

Solution:

The author compares the trilateral strategic consultations among the United States, India and Japan to the Franco-BritishRussian Triple Entente formed prior to World War I. This entente was formed to ...meet the threat posed by the rapid rise
of an increasingly assertive Germany.. Thus, the paragraph must conclude by telling us how the author might justify his/her
comparison - by mentioning the modern counterpart of historical Germany. Only option 4 provides the required comparison.
The author has not mentioned the three democracies (U.S.-India-Japan) as the entente (the paragraph only mentions the
efforts of the three countries in trying to form an entente, it does not state that an entente already exists), hence option 1
though somewhat right is too premature for this paragraph.
The paragraph does not provide enough information to substantiate the claim made in option 2.
The paragraph builds a case for a strategic understanding between three countries, without focusing upon the specific role of
each country. Thus, option 3 if considered, will bring in a new point into the discussion and not complete the paragraph in a
logical manner.
Hence, the correct answer is option 4.

71.

Fill in the blanks with the most appropriate set of words from the given options.

3 Marks

The transition of China in the last ten years from a country known for its ________ to an extremely tourist-friendly one has roots both in
the perceived commercial possibilities, and in the inherent ________ , where in the eyes of the world the Chinese simply have to appear
the best and brightest.
1)

forbearance, masochism

2)

melanophobia, chauvinism

3)

xenophobia, chauvinism

4)

anti-elitism, masochism

Solution:
The term xenophobia refers to hatred for foreigners; among its other meanings chauvinism also means undue partiality or
attachment to a group or place to which one belongs. When these vocabulary constrains are overcome, it is easy to see how the context
accommodates both these words well. Transition from xenophobia to tourist-friendliness; a desire to appear to be the best due to an
inherent chauvinism.
The meanings of the other words are as follows:
Forbearance means patience. The statement does not delve into this aspect as far as China is concerned. Eliminate option 1.
Melanophobia is a hatred for the color black. Eliminate option 2.
Masochism refers to the tendency to find pleasure in self-denial, submissiveness, etc.. This term does not fit the context of the
Chinese having to appear as the best and the brightest. Eliminate option 4.
Hence, the correct answer is option 3.
Group Question
Answer the following question based on the information given below.
The path diagram below shows the distance between the various money transmission offices in a city. The values in percentage indicate
percentage of the total amount that is charged by the transmission office. There is no charge from the starting and destination office. There is
also a transmission charge of n% of the total amount for the distance between starting and destination office is n km. The trasferred amount

http://testfunda.com/LMS/Student/NewReports.aspx

33/47

reaches destination office after deducting the abovesaid charges.


Note: 2 along the path joining A and B indicates that the distance between A and B is 2 km.

72.

3 Marks

What is the maximum amount that will reach office H if Rs. 500 are transferred from office A provided that no more than two
intermediary offices are covered during the transit?
1)

Rs.475.50

2)

Rs.432.50

3)

Rs.482.50

4)

Rs.452.50

Solution:
The ways in which transmission can be done from A to H having maximum of two intermediary offices are:
Way 1: A C I H
Way 2: A D I H
Way 3: A D F H
The transmission charges for each way are:
Way 1: 3% of 500 + 2% of 500 + 11% of 500 = 16% of 500
Way 2: 0.5% of 500 + 2% of 500 + 11% of 500 = 13.5% of 500
Way 3: 0.5% of 500 + 2% of 500 + 7% of 500 = 9.5% of 500
So, the maximum amount reaching H is be through Way 3, which is 90.5% of 500 = Rs.452.50
Hence, option 4.
73.

3 Marks

Find the number of ways to transmit money from office D to E having exactly three intermediary offices.
1)

10

2)

12

3)

13

4)

14

Solution:
The ways in which transmission can be done from D to E are:
Way 1: D F H I E
Way 2: D F I G E

http://testfunda.com/LMS/Student/NewReports.aspx

34/47

Way 3: D F I C E
Way 4: D I C G E
Way 5: D I G C E
Way 6: D I C B E
Way 7: D C I G E
Way 8: D C G I E
Way 9: D C A B E
Way 10: D A C B E
Way 11: D A C G E
Way 12: D A C I E
Way 13: D A B C E
Thus, there are 13 ways satisfying the given condition.
Hence, option 3.
74.

What is the maximum distance which would be required to cover to transmit money from office D to E having exactly three
intermediary offices?

3 Marks

1)

27 km

2)

15 km

3)

32 km

4)

18 km

Solution:
The maximum distance will be covered when the way chosen is D I C G E as it has the stations which are very far from
each other.
Distance covered using this way = 8 + 7 + 9 + 3 = 27 km
Hence, option 1.
75.

Answer the question based on the passage given below.

3 Marks

Incandescent bulbs produce mostly heat, which is why a fluorescent using only 13 watts of electricity can produce light comparable to an
incandescent hogging 60 watts. Each 13-watt CFL (Compact fluorescent lamps), over the expected 10,000 hour life of the bulbs, will
save 470 kilowatt-hours (kWh) of electricity as compared to 60-watt bulbs. This translates to a global warming-fighting reduction of over
730 pounds of carbon dioxide. It also means a reduction of 1.6 pounds of nitrogen oxides (which contribute to ozone and acid rain) and
4.3 pounds of sulfur dioxide (which contributes to haze and acid rain). CFL must become the only choice for all lighting purposes.
All the following can be advanced in favor of the above argument EXCEPT?
1)

CFL bulbs have been known to last twice as much on Solar power as compared to incandescent bulbs.

2)

The useful life hours for similar CFL and incandescent bulbs are 10,000 and 1,500 hours respectively.

3)

Electricity used in kilowatt hours by CFL and incandescent bulbs are 130 and 600 respectively.

4)

Bulb costs for CFL and incandescent are Rs. 110 and Rs. 12 each respectively.

Solution:
The arguments in options 1, 2 and 3 are in favour of CFL bulbs, whereas the question stem requires an argument which is not in favour
of CFL bulbs. Therefore, options 1, 2 and 3 can be eliminated.
In option 4, the price comparison is not in favour of CFL bulbs. If the cost comparison was done with the inclusion of electricity costs,
CFL would still have had an upper-hand. Option 4 however, only compares the bulb costs between CFL and incandescent bulbs.
Hence, the correct answer is option 4.
Group Question
The passage given below is followed by a set of questions. Choose the most appropriate answer to each question.
A century of debate over the social and aesthetic value of culture has been hollowed out. Not many observers of our commodity-based
system of production pretend to care any longer about the dilemmas of art in a democratic society. There seems no point in questioning
whether a painting, book, or film is good or bad. A society that has lost hope of seeing itself reflected in its culture naturally loses interest in it.
The erasure of the legacy of the workers strike demanding the right to pursue a culture of their own choosing in Lawrence bespeaks a definite
migration of cultural power from the redoubts of society and government to unaccountable, unrepresentative, and inaccessible agencies of
resource allocation : velvet-gloved foundations and debt-producing colleges and universities, mainly. The results have been boring and
unreal, a culture for nobodys sake, at once arbitrary and overdetermined. In the foundations and universities, as in the corporate marketing
departments from which they borrow their strange notions, a class-specific fetish for creativity coincides with an invincible belief in

http://testfunda.com/LMS/Student/NewReports.aspx

35/47

meritocracy, while cartel-like techniques of managed competition muffle the contradiction. Americas stagnation proceeds directly from the
assumption that cultural activity requires only enough funding to generate ratings, credentials, prizes, and tourist dollars. The managers
ensure that nothing too interesting, idiosyncratic, or passionate reaches the public.
This brings you to a roll call of the inert, sterile, and depraved cultural leavings of our plutocratic age. Welcome to an America that offers up
neither bread nor roses, but a thin philanthropic gruel that advertises the baronial status of business, and a luxury-grade higher education that
emits a boosterish fog. You will read here of decomposing cities that glitter with vibrancy, TV moguls who stage fables of competitive
individualism, and Very Serious novelists chasing Very Important literary prizes.
76.

3 Marks

Which of the following best articulates the central idea of the passage?
1)

The devaluation of artistic merit has been a consequence of Americas deep-rooted elitism.

2)

The rise of materialism in America has coincided with a dilution of its culture.

3)

The sorry state of cultural pursuits in America has been a result of the waning of its once-prestigious educational
institutes.

4)

The increased importance given to wealth has led to a decline in the authenticity of culture.

Solution:
The passage presents a critique of Americas culture in the present day and age and how the increasing importance given to
materialism has contributed to the same. This is encapsulated in Americas stagnation proceeds directly from the assumption that
cultural activity requires only enough funding to generate ratings, credentials, prizes, and tourist dollars.. This points to option 4
being the correct option.
The passage does discuss elitism in America but it does not delve deeper into its history; instead it concerns itself with the
consequences of this elitism. Eliminate option 1.
Option 2 distorts the relationship between materialism and cultural decline in America. The passage establishes the strong
correlation between the two which is not limited to being resultant and hence, cannot be termed as a coincident relationship.
While the passage does detail on the decline of educational institutions in America, they do not comprise its central idea.
Eliminate option 3.
Hence, the correct answer is option 4.
77.

3 Marks

The passage cites all of the following as reasons for Americas cultural stagnation except:
1)

A stifling of expression to pander to the whims of the populace.

2)

Creation of works which conform to an ideal dictated by class but have little artistic value

3)

A lack of representation of different social classes in Americas cultural diet.

4)

The sidelining of artistic merit in favour of the ability to generate funds.

Solution:
Option 2 can be inferred from ...as in the corporate marketing departments from which they borrow their strange notions, a classspecific fetish for creativity coincides with an invincible belief in meritocracy, while cartel-like techniques of managed competition
muffle the contradiction..
Option 3 can be deduced from A society that has lost hope of seeing itself reflected in its culture naturally loses interest in it..
Note that representation of social classes is not the same as conforming to an ideal dictated by class. All classes will not be
represented in acts that conform to social ideals.
According to the passage, ... cultural activity requires only enough funding to generate ratings, credentials, prizes, and tourist
dollars. The managers ensure that nothing too interesting, idiosyncratic, or passionate reaches the public.. This establishes the
inference mentioned in option 4 and hence, eliminates the same.
Option 1 alone contradicts the information given in the passage which states The erasure of the legacy of the workers strike
demanding the right to pursue a culture of their own choosing in Lawrence bespeaks a definite migration of cultural power from
the redoubts of society and government to unaccountable, unrepresentative, and inaccessible agencies of resource allocation :
velvet-gloved foundations and debt-producing colleges and universities, mainly.. Thus, the preferences of the common lot are far
from being pandered to. This vindicates option 1.
Hence, the correct answer is option 1.

78.

3 Marks

In accordance with the line of thought pursued in the passage, which of the following is most likely to occur in modern day
America?
1)

A book depicting the lives of the elite becomes a bestseller among blue collar workers.

2)

An artist becomes popular after her paintings are bought by a billionaire.

3)

A musician struggles to release his compositions due to his personal lifestyle choices.

4)

A playwright is criticized for his modern day parody of a classic play.

Solution:
The passage mentions ...an America that offers up neither bread nor roses, but a thin philanthropic gruel that advertises the
baronial status of business, and a luxury-grade higher education that emits a boosterish fog. You will read here of decomposing
cities that glitter with vibrancy, TV moguls who stage fables of competitive individualism, and Very Serious novelists chasing
Very Important literary prizes.. This lays emphasis on the elitist tendencies that underlie Americas cultural stride. Moreover, the
passage describes Americas cultural leanings as ... inert, sterile, and depraved .... We can thus, infer that any attempt to defy
the conventions set by the elite would be regarded as disrespectful and criticized in the social scenario described by the passage.
Therefore, a playwright challenging the prevalent perspective on a play, would certainly be criticized. This vindicates option 4.
Option 1 describes a fascination about the lives of the elite among those who do not have access to similar lifestyles; this has not

http://testfunda.com/LMS/Student/NewReports.aspx

36/47

been dealt with in the passage.


While option 2 may seem tempting, the passage does not establish the relationship between the mere possession of material
wealth and the ability to influence the cultural paradigm. It attributes this ability to educational institutes and business houses
which deem what could generate adequate funds.
The passage does not discuss the influence of artists personal lifestyle choices on his/her art. Eliminate option 3.
Hence, the correct answer is option 4.
Fill in the blanks with the most appropriate pair of words from the given options.

79.

3 Marks

When urbanism reaches a particular pitch of intensity and remains there for decades uninterrupted by _____ or blank-slate utopianism,
we find the _____ of successive waves of technology.
1)

amelioration, effect

2)

catastrophe, encrustation

3)

disaster, epitome

4)

thaumaturgy, blight

Solution:
The statement mentions the prolonged effect of urbanism on a particular space. The first blank must indicate an entity which could
interrupt the successive waves of technology mentioned in the second part of the statement. This eliminates option 1 with
amelioration meaning betterment as it does not indicate a change of colossal magnitude.
Catastrophe, disaster and thaumaturgy which means magic; working of miracles and can be used for the first blank.
For the second blank, we need a word which will describe the impact of successive waves of technology on urbanism. Since the
statement implies a sense of balance between technological change and urbanism, option 4 with blight meaning any cause of
destruction, impairment or ruin cannot be used for the second blank.
Epitome refers to 'a thing that is typical of or possesses to a high degree the features of a whole class'. The word epitome denotes a
superlative point in a time frame. Thus, it does not go with the pitch of intensity in which urbanism remains constant for decades without
being interrupted. Moreover, the usage of successive waves of technology denotes the presence of several layers and not a particular
high point. Eliminate option 3.
Thus, option 2 with catastrophe and encrustation meaning covering or coating on the surface of a body is the most suitable option.
Hence, the correct answer is option 2.

80.

3 Marks

The question below consists of a set of labelled sentences. These sentences, when properly sequenced, form a coherent paragraph.
Choose the most logical order of sentences from the options.
I. Although population genetics is still a young science, it seems to indicate that a significant proportion of these peoples' ancestry
comes from a common Near Eastern population to which (despite the differences with the Biblical genealogy) the term "Semitic"
has been applied.
II. Modern science identifies a population's common physical descent through genetic research, and analysis of the Semiticspeaking peoples suggests that they have some common ancestry.
III. However, this correlation should rather be attributed to said common Near Eastern origin, as for example Semitic-speaking Near
Easterners from the Fertile Crescent are generally more closely related to non-Semitic speaking Near Easterners, such as
Iranians, Anatolians, and Caucasians, than to other Semitic-speakers, such as Gulf Arabs, Ethiopian Semites, and North African
Arabs.
IV. Though no significant common mitochondrial results have been yielded, Y-chromosomal links between Semitic-speaking NearEastern peoples like Arabs, Assyrians and Jews have proved fruitful, despite differences contributed from other groups.
V. All Asian peoples were thought of as descendants of Shem and the term Semitic was confined to the ethnic groups who have
historically spoken Semitic languages who were often considered to be a distinct race.
1)

V, II, IV, I, III

2)

II, I, III, IV, V

3)

II, IV, III, V, I

4)

V, IV, III, II, I

Solution:
The sequence pertains to the people who share ancestry in the Semitic race. Between the two options provided as sequence starters
statement II and statement V, statement V makes for a better introductory statement. It puts forth a hypothesis about the Semitic race
and highlights the historical context associated with it. Statement II then confirms the hypothesis by delving into the aspect of how
genetic research can help in identifying a population's common physical descent. Eliminate options 2 and 3.
Thus, statements V and II form a logical pair. These two statements are quite far apart in option 4. We can eliminate option 4.
statements IV and I also form a pair. Both are about population genetics confirming the Semitic race. Statement IV mentions

who the Near-Eastern peoples are while I mentions what is common between them. Statement III concludes the sequence
by highlighting the influence of geography on ancestry. Thus, the most logical sequence is V, II, IV, I, III.
Hence, the correct answer is option 1.
Group Question
Answer the following question based on the information given below.
Three couples gather for a birthday party. These couples consist of three women Asmita, Boby, Celine; and three men Dinesh, Eshaan,
Furqaan. They sit around a rectangular table as shown below, such that everyone is facing the centre.

http://testfunda.com/LMS/Student/NewReports.aspx

37/47

The following information is also known;


1.
2.
3.
4.

81.

Asmita sits opposite Boby or Dinesh.


Eshaan sits to the immediately left of Celine.
Furqaan sits to the immediate left of one woman and to the immediate right of another woman.
The person whose birthday is being celebrated is the only person who sits opposite a man and to the immediate left of a woman.

3 Marks

Whose birthday is being celebrated?


1)

Boby

2)

Celine

3)

Furqaan

4)

Cannot be determined

Solution:
Let the person whose birthday is being celebrated be denoted by t.
Let Asmita, Boby, Celine, Dinesh, Eshaan and Furqaan be denoted by A, B, C, D, E and F respectively.
Let the woman who is to the immediate right of t be W1 and the man opposite t be M1.
Hence, we have the following possible arrangements;

The other three positions are simply mirror images of these three positions.
Here, x, y and z are the three other people.
Now, consider each combination separately.
Case I:
In this case, if x is a man, then y should also be a man; otherwise zs position will contradict the fourth condition.
Similarly, if x is woman then y must also be a woman; otherwise W1s position will contradict the fourth statements.
Hence, in this case, we get two possible arrangements.

Consider Case Ia;


By, third statement, M3 = F. So, by the second statement, M1 is E and W2 is C.
Hence, the M2 must be D.
Hence, we have following arrangement.

http://testfunda.com/LMS/Student/NewReports.aspx

38/47

Here, t and W1 are A and B in no particular order.


But this doesnt satisfy the first condition. Hence it is not possible.
Consider Case Ib;
In this case also, by third statement, t is F, and by second statement, M2 is E and W3 is C.
Hence, M1 must be D.
Hence, we have the following arrangement;

Here, again, W1 and W3 are A and B in no particular order.


But this doesnt satisfy the first condition. Hence it is not possible.
Case II:
In this case, if x is a man, then then t must be man; otherwise ys position will contradict the fourth condition. Similarly, if x is
woman, then t must also be woman; otherwise M1s position will contradict fourth condition.
Hence, we have the following arrangements;

Consider case IIa;


By third statement, t is F, and by second statement M2 is E and W3 is C.
Hence, M1 is D.
Here W1 and W2 are A and B in no particular order.
Hence, we have the following arrangement;

But, this cannot satisfy the first condition.


Hence this case is not possible.
Consider case IIb;
By, third condition, M1 is F.
By statement 2, M2 is E and t is C.
Hence M3 must be D.
Hence, we have the following arrangements.
By condition 1, W1 is A and W2 is B. In this case, we get the following arrangement;

http://testfunda.com/LMS/Student/NewReports.aspx

39/47

This is the possible arrangement.


Case III:
Note: x cannot be a man. As if y is a woman, zs position will contradict the fourth statement. But if x and y are man, then z must
be woman. But, in this case, W1s position contradicts statement 4.
Similarly, if x is a woman then t must also be a woman; otherwise M1s position will contradict statement 4. But if x and t are
woman, then y and z must be a man. But this contradicts statement 3.
Hence, Case III is not possible.
So, the only possible arrangement possible is case IIb.
Hence, final arrangement is;

Hence, Celines birthday is being celebrated.


Hence, option 2.

82.

Who sits to the immediate right of Asmita?

3 Marks

1)

Dinesh

2)

Celine

3)

Furqaan

4)

Cannot be determined

Solution:
From the solution to the first question of the set; Furqaan sits to the immediate right of Asmita.
Hence, option 3.
83.

Who among the following are the immediate neighbours of Furqaan?

3 Marks

1)

Asmita and Boby

2)

Asmita and Celine

3)

Boby and Celine

4)

Cannot be determined

Solution:
From the solution to the first question of the set; Asmita and Boby are the immediate neighbours of Furqaan.
Hence, option 1.
84.

3 Marks

A base word has been used in the options given below. Choose the option in which the usage of the word is incorrect or inappropriate.
Bowl
1)

His family was bowled over by the news.

2)

Australia was bowled out for 133 in the T20 world cup final against England.

3)

The recipe requires you to mix all ingredients in a shallow bowl.

4)

He ended the contest with a quickly bowled left hook.

http://testfunda.com/LMS/Student/NewReports.aspx

40/47

Solution:
To be bowled over means to be greatly surprised. It has been used correctly in option 1.
To be bowled out means to be retired, fall of wickets in the game of cricket. It has been used correctly in option 2.
A shallow bowl is a deep round container used chiefly for holding liquids. The usage in option 3 is correct.
In the case of option 4, the sentence refers to a boxing match and the term - a left hook, is used correctly with the verb throw and not
bowled.
Hence, the correct answer is option 4.
The following question consists of a certain number of sentences. Select the sentences that are grammatically incorrect or inappropriate.

85.
3 Marks

A.
B.
C.
D.

Audiences are able to relate to the characters much better in such settings.
I think many of us are super fed up with stupid dream sequences happening
in the most unlikely of stories. Inserting brainless fantasies into a movie show
a lack of confidence on the director and I am glad this movie has none of that.

1)

A and B

2)

C and D

3)

A, C and D

4)

B and D

Solution:
Statement C has subject verb agreement error. It should be Inserting brainless fantasies into a movie shows since shows applies
to a lack of confidence. Eliminate options 1 and 4.
Statement D has a prepositional error. A lack of confidence on the part of the director or a lack of confidence of the director would
rectify it.
Statements A and B are grammatically correct. Eliminate option 3.
Hence, the correct answer is option 2.
86.

3 Marks

The question below consists of a set of labelled sentences. These sentences, when properly sequenced, form a coherent paragraph.
Choose the most logical order of sentences from the options.
I.
II.
III.
IV.

On the table in front of Sam stood a glass and an open bottle of champagne.
Richard entered the tavern.
Inside the room he also found a girl with a flute, a good looking rosy cheeked girl of 19, wearing a striped skirt with ribbons.
He found Sam in a small backroom, wherein clerks, merchants and numerous other people were drinking coffee to the bawling of
singers.
V. In spite of the singers in the room, she was singing a hall song in a husky voice to the accompaniment of a flute.
1)

I, II, IV, III, V

2)

IV, I, II, III, V

3)

II, IV, I, III, V

4)

II, I, IV, V, III

Solution:

All of the statements discuss what went on inside the tavern. The other statements logically state what Richard saw as he
entered the tavern. Eliminate options 1 and 2.
Statement IV comes next in the sequence as the pronoun he in it addresses Richard. Eliminate option 4.
Statement I follows statement IV because it describes what Sam, who is introduced in statement IV was doing.
Statement III follows statement I because it contains the pronoun he which ascribes to Richard and the word also
indicates that the girl was seen after Sam. First he noticed Sam and then the girl.
The last in the sequence is statement V because it contains the pronoun she referring to the rosy cheeked girl in III.
Hence, the correct answer is option 3.
Group Question
Answer the following question based on the information given below.
There are 7 consecutive odd numbers represented by A to G in any order. The following details are known about them
a.
b.
c.
d.
e.
f.

87.

The average of all the numbers is 67.


There are 4 prime numbers in the list and they are represented by A, C, D and G in any order.
D+C=A+E
B is the only number divisible by 5.
G is closest to the average.
C is the greatest prime number of the 7 numbers.

3 Marks

What is the value of A?

http://testfunda.com/LMS/Student/NewReports.aspx

41/47

1)

61

2)

67

3)

73

4)

71

Solution:
Since the given numbers are consecutive odd numbers, they can be said to range from n 6 to n + 6 such that n is the central
number.
Thus, the sum of these 7 numbers becomes 7n.
Since the average of these 7 numbers is 67, their sum is 67 7 = 469.
7n = 469
n = 67
Thus the numbers are 61, 63, 65, 67, 69, 71 and 73.
Since G is the closest to the average, G = 67.
Out of these 61, 67, 71 and 73 are prime numbers.
Since G = 67, A, D and G are 61, 71 and 73 (in no specific order).
Since C is the greatest prime number, C = 73.
Since B is divisible by 5, B = 65.
Now the remaining numbers are 61, 63, 69 and 71.
We have D + C = A + E
73 + D = A + E
Now D and A can be 61 or 71 (in any order).
Case 1: D = 71
A = 61
E = 73 + 71 61 = 83
However, the largest number in the group is 73.
Therefore, this case is not possible.
Case 2: D = 61
A = 71
E = 73 + 61 71 = 63
F = 69
Hence, option 4.

88.

3 Marks

How many factors does E have?


1)

2)

3)

4)

Solution:
E = 63 = 71 32.
Thus, number of factors of E is (1 + 1) (2 + 1) = 2 3 = 6
Hence, option 2.

89.

3 Marks

What is the value of A + C + E + G equal to?


1)

274

http://testfunda.com/LMS/Student/NewReports.aspx

42/47

2)

272

3)

278

4)

276

Solution:
A + C + E + G = 71 + 73 + 63 + 67 = 274
Hence, option 1.
Group Question
Answer the following question based on the information given below.
In a remote jungle of the Amazon, there is a small village called Hip-Hop. In this village, the residents belong to exactly one of two tribes: the
tribe that speaks truth about whatever they believe in (Hip) or the tribe that always speaks in contradiction to their own beliefs (Hop). Further,
within each of the tribes, there are two more categories of people: one group whose every belief is right (Tip), and the other whose every
belief is actually wrong (Top).
Andy, Brandi, Cindy and Dandy are the only residents of this village. The following is the transcript of one of their conversations:
Andy: Im Top. Cindy is Hip.
Brandi: Im Hip. Dandy is Top.
Cindy: Im Hop. Brandi is Hop.
Dandy: Im Tip. Cindy is Tip.

90.

3 Marks

Who among the following is Hip-Tip?


1)

Andy

2)

Brandi

3)

Cindy

4)

Cannot be determined

Solution:
We have four categories of people:
Hip-Tip
Hip-Top
Hop-Tip
Hop-Top
Now, let us find out what each one of these category of people will say about themselves:
Hip-Tip : Im Hip-Tip.
Hip-Top : Im Hop-Tip.
Hop-Tip : Im Hip-Top.
Hop-Top : Im Hop-Top.
A Hop-Top is actually lying about a lie, and is therefore actually speaking the truth.
Now, consider the first statement made by Andy, Im Top.
Based on above four probable statements which could be made by the people belonging to any of the above category, Andy
belongs to one of the two categories i.e. Hop-Tip and Hop-Top.
But this statement has to necessarily come from a person belonging to Hop category.
Andy is necessarily Hop.
Similarly using the same logic for others, it becomes evident that:
Brandi is Tip, Cindy is Top and Dandy is Hip.
Consider the last statement made by Dandy, Cindy is Tip.
We know that Dandy is Hip, it implies that he is speaking the truth and he believes Cindy to be a Tip. However we also know
that Cindy is Top.
It implies that the belief of Dandy is wrong.
Dandy must be a Top.
Dandy is Hip-Top.
Further, Brandi says that Dandy is Top, but now we know that Dandy is actually Top.

http://testfunda.com/LMS/Student/NewReports.aspx

43/47

The statement made by Brandi about Dandy is correct.


Also Brandi is Tip, which implies that both the statements of Brandi are correct, leading to the conclusion that Brandi is Hip-Tip.
Consider the statement made by Cindy, Brandy is Hop.
We now know that Brandi is Hip, it implies that the statement made by Cindy is wrong.
Also further we know that Cindy is Top. Now if Cindy were to belong to Hop, then Cindy would be actually lying about a lie and
hence would be telling a truth.
It can be inferred that Cindy is Hip.
Consider the statement made by Andy, Cindy is Hip. which is a true statement.
However, we know that Andy is Hop.
It can be inferred that Andy is lying about a lie leading to a correct statement.
It can be inferred that Andy is Hop-Top.
All this information gives us the following:
Andy: Hop-Top
Brandi: Hip-Tip
Cindy: Hip-Top
Dandy: Hip-Top
As can be seen, Brandi is Hip-Tip.
Hence, option 2.

91.

The number of Hip people among these four exceeds the number of Hop people by what percentage?

3 Marks

1)

100

2)

200

3)

50

4)

Solution:
The number of Hip people is 3 and that of Hop people is 1.

Hence, option 2.
92.

Find the difference between the number of the Top and the Tip people among the four.

3 Marks

1)

2)

3)

4)

Solution:
The number of Top people is 3 and the number of Tip people is 1.
The required answer is (3 1) = 2
Hence, option 3.
93.

3 Marks

Fill in the blanks with the most appropriate pair of words from the given options.
Watson describes Holmes as "________" in habits and lifestyle. According to Watson, Holmes is an _______, with no regard for
contemporary standards of tidiness or good order.
1)

behemoth, agnostic

2)

abrasive, ecclesiastic

3)

dandy, avarice

4)

bohemian, eccentric

Solution:

http://testfunda.com/LMS/Student/NewReports.aspx

44/47

The clue lies in the phrase ...no regard for contemporary standards of tidiness..., which implies that Holmes must have been a man who
was untidy and disorderly in his habits and that they were deviant from contemporary standards. The words that come closest to
describe this trait are bohemian which means having or displaying a very informal and unconventional way of life and eccentric,
which means not conventional or normal.
None of the other options are suitable to the context.
Hence, the correct answer is option 4.
94.

3 Marks

The question below consists of a set of labelled sentences. These sentences, when properly sequenced, form a coherent paragraph.
Choose the most logical order of sentences from the options.
I. From the Banks establishment until today, the unwritten rule has been that the US government simply designates each new
president: all 11 have been Americans, and not a single one has been an expert in economic development, or had a career in
fighting poverty or promoting environmental sustainability.
II. With the Bank just two blocks away from the White House on Pennsylvania Avenue, it has been all too easy for the US to
dominate the World Bank.
III. Instead, the US has selected Wall Street bankers and politicians, presumably to ensure that the Banks policies are suitably
friendly to US commercial and political interests.
IV. Now many members, including Brazil, China, India, and several African countries, are raising their voices in support of more
collegial leadership and an improved strategy that works for all.
1)

I, II, IV, III

2)

I, III, II, IV

3)

II, IV, I, III

4)

II, I, IV, III

Solution:
Of the four sentences only statement II mentions the World Bank in full, this should help us identify it as the introductory sentence. If one
misses this it becomes difficult to move on. Eliminate options 1 and 2. Though, I, III, II, IV would make sense, the reader would not know
which bank is being discussed until much later.
The end of statement I and beginning of statement III connect logically. Hence, statements I and III must be placed together. Option 4
misses out on doing this.
Hence, II, IV, I, III makes for the most logical sequence.
Hence, the correct answer is option 3.
Group Question
The passage given below is followed by a set of questions. Choose the most appropriate answer to each question.
Periodic large-scale mass extinctions have occurred throughout the history of life; indeed, it is on this basis that the geologic eras were first
established. Of the five major mass extinction events, the one best known is the last, which took place at the end of the Cretaceous Period
and killed the dinosaurs. However, the largest of all extinction events occurred between the Permian and Triassic periods at the end of the
Paleozoic Era, and it is this third mass extinction that profoundly affected life during the Triassic. The fourth episode of mass extinction
occurred at the end of the Triassic, drastically reducing some marine and terrestrial groups, such as ammonoids, mammal-like reptiles, and
primitive amphibians, but not affecting others. Though the Permian-Triassic mass extinction was the most extensive in the history of life on
Earth, it should be noted that many groups were showing evidence of a gradual decline long before the end of the Paleozoic. Nevertheless,
85 to 95 percent of marine invertebrate species became extinct at the end of the Permian. On land, four-legged vertebrates and plants
suffered significant reductions in diversity across the Permian-Triassic boundary. Only 30 percent of terrestrial vertebrate genera survived into
the Triassic. Many possible causes have been advanced to account for these extinctions. Some researchers believe that there is a periodicity
to mass extinctions, which suggests a common, perhaps astronomical, cause. Others maintain that each extinction event is unique in itself.
Cataclysmic events, such as intense volcanic activity and the impact of a celestial body; or more gradual trends, such as changes in sea
levels, oceanic temperature, salinity, or nutrients, fluctuations in oxygen and carbon dioxide levels, climatic cooling, and cosmic radiation,
have been proposed to explain the Permian-Triassic crisis. Unlike the end-Cretaceous event, there is no consistent evidence in rocks at the
Permian-Triassic boundary to support an asteroid impact hypothesis, such as an anomalous presence of iridium and associated shocked
quartz. A more plausible theory is suggested by finely laminated pyritic shales, rich in organic carbon, that are commonly found at the
Permian-Triassic transition in many areas. These shales may reflect oceanic anoxia in both low and high latitudes over a wide range of shelf
depths, perhaps caused by weakening of oceanic circulation. Such anoxia could devastate marine life, particularly the bottom-dwellers. Any
theory, however, must take into account that not all groups were affected to the same extent by the extinctions.

95.

3 Marks

Researchers hypothesize that the Permian-Triassic mass extinction was


A.
B.
C.
D.

not caused by cataclysmic events.


not caused by asteroid impact.
caused by oceanic anoxia.
caused by periodicity of mass extinctions.

1)

A, B and C

2)

A, C and D

3)

B, C and D

4)

All of the above

Solution:
According to the passage, cataclysmic events include volcanic activity and the impact caused by a celestial body. The passage
does not support the asteroid impact hypothesis. The passage states that Cataclysmic events, such as intense volcanic activity
and the impact of a celestial body; or more gradual trends ... have been proposed to explain the Permian-Triassic crisis.. Though

http://testfunda.com/LMS/Student/NewReports.aspx

45/47

volcanic activities and other cataclysmic events (if any) are not ruled out; given that the asteroid impact is ruled out, statement A
does not entirely hold. Eliminate options 1, 2 and 4.
According to the passage, ...there is no consistent evidence in rocks at the Permian-Triassic boundary to support an asteroid
impact hypothesis.... This vindicates statement B.
According to the passage, A more plausible theory...oceanic anoxia...anoxia could devastate marine life, particularly the bottomdwellers. Any theory, however, must take into account that not all groups were affected to the same extent... and Some
researchers believe that there is a periodicity to mass extinctions, which suggests a common, perhaps astronomical, cause.. This
vindicates statements C and D.
Therefore the correct combination is B, C and D.
Hence, the correct answer is option 3.
96.

According to the passage, ... an anomalous presence of iridium and associated shocked quartz in geologic rocks would indicate
which of the following?

3 Marks

1)

An asteroid impact.

2)

Mass extinctions.

3)

Weakening of oceanic circulation.

4)

Periodicity of mass extinctions.

Solution:
The passage states, Unlike the end-Cretaceous event, there is no consistent evidence in rocks at the Permian-Triassic boundary
to support an asteroid impact hypothesis, such as an anomalous presence of iridium and associated shocked quartz.. This
supports option 1. The presence of iridium and shocked quartz signify an asteroid impact.
In light of the above extract, we can eliminate options 2, 3 and 4.
Hence, the correct answer is option 1.
97.

According to the passage, which of the following is true about the fourth mass extinction that took place?

3 Marks

1)

It happened at the end of the Cretaceous Period and killed the dinosaurs.

2)

It observed the extinction of 85 to 95 percent of marine invertebrate species.

3)

It observed the extinction of 70 percent of terrestrial vertebrate genera.

4)

Not all groups of creatures were affected to the same extent by the extinction.

Solution:
According to the passage, The fourth episode of mass extinction occurred at the end of the Triassic, drastically reducing some
marine and terrestrial groups, such as ammonoids, mammal-like reptiles, and primitive amphibians, but not affecting others.. This
vindicates option 4 as options 1, 2 and 3 relate to different extinctions.
The fourth extinction was one that happened at the end of the Triassic, which was followed by the fifth that wiped out the
dinosaurs.
The third was the Permian-Triassic in which 85 to 95 percent of marine invertebrates became extinct, as well as 70 percent of
terrestrial vertebrate.
In geological time - the Paleozoic Era ended with the Permian period.
Hence, the correct answer is option 4.
98.

3 Marks

The following question consists of a certain number of sentences. Some sentences are grammatically incorrect or inappropriate. Select
the option that indicates the grammatically correct and appropriate sentence(s).
A.
B.
C.
D.

Most of the military disasters have been caused by the home government.
Napoleon undoubtedly owed much of his extraordinary success to his goodwill.
Hence the saying: If you know the enemy and know you, you need not fear the result of a hundred battles.
If you know yourself but not the enemy, for every victory gained one will also suffer a defeat.

1)

A, B and C

2)

B, C and D

3)

C and D

4)

A and B

Solution:
Statement C should have the pronoun you (second one) in the emphatic case. Simply put, the word should be yourself as it reflects on
the person you. Eliminate options 1, 2 and 3.
In statement D, for pronoun consistency, the word one should be replaced by the pronoun you.
Statements A and B are grammatically correct.
Hence, the correct answer is option 4.
99.

3 Marks

The following question has a paragraph from which the last sentence has been deleted. From the given options, choose the one that
completes the paragraph in the most appropriate way.
In Advertising, you have to have the courage to stand up for what you believe in. No matter the opposition. Play it from your heart,
always. Play on the front foot. Believe in your idea and be honest to yourself that you have given the best. Then go all out and present
the idea. Laugh. Laugh. Laugh. A remedy for everything from a bad brief to NO idea - laugh. The more you laugh the better life will
be._______________________________.

http://testfunda.com/LMS/Student/NewReports.aspx

46/47

1)

Stop taking everything so seriously.

2)

Believe in relationships, partnerships and friendships.

3)

And communicate from your heart.

4)

When people have fun at work, great ideas come by.

Solution:
With the laugh, laugh etc. for the penultimate sentence option 1 and 4 are the ones to be evaluated seriously. The other options are
a direct address to 'you'.
Option 2 is too broad.
Option 3 with And is not apt.
Option 1 continues the idea presented and helps close the paragraph.
Though option 4 may seem to continue the line of thought presented in the paragraph, it is not as suitable as option 1. Option 4 talks
about having fun at work where as the penultimate sentence talks about life.
Hence, the correct answer is option 1.

100.

3 Marks

Four sentences numbered I, II, III and IV are given in random order. Three of these sentences belong to the same paragraph, and can be
arranged logically to form a coherent paragraph. One of the sentences does not fit into this paragraph. Choose the ODD sentence for
your answer.
I. So, while the hikes for both fuels move in tandem, the ratio of difference between them remains more or less the same.
II. Today, diesel engines offer good performance, excellent mileage and, on many counts, diesel is considered the cleaner fuel.
III. No matter which way the prices are headed, one fact is unlikely to change anytime soon in the country: diesel is still cheaper than
petrol, thanks to the subsidy that the former enjoys.
IV. On an average, this difference is Rs 25.
1)

2)

II

3)

III

4)

IV

Solution:
Statements III, I and IV in that order offer a discussion on the price difference between diesel and petrol in India owing to subsidies. The
cleaner fuel part mentioned in statement II is unrelated to this discussion.
Hence, the correct answer is option 2.

http://testfunda.com/LMS/Student/NewReports.aspx

47/47

You might also like